Bookmark and Share

ICWAI Paper-5 Financial Accounting REVISIONARY TEST PAPER(RTP) for INTERMEDIATE DECEMBER 2009 TERM OF EXAM

Paper-5 Financial Accounting REVISIONARY TEST PAPER(RTP) for INTERMEDIATE DECEMBER 2009 TERM OF EXAMINATION
DOWNLOAD

INTERMEDIATE EXAMINATION
(REVISED SYLLABUS - 2008)
GROUP - I
Paper-5 : FINANCIAL ACCOUNTING
Q1. State with reasons whether the following are Capital Expenditure or Revenue Expenditure:
(i) Expenses incurred in connection with obtaining a licence for starting the factory were Rs
10,000.
(ii) Rs 1,000 paid for removal of stock to a new site.
(iii) Rings and Pistons of an engine were changed at a cost of Rs 5,000 to get full efficiency.
(iv) Rs 2,000 spent as lawyer¡¯s fee to defend a suit claiming that the firm¡¯s factory site
belonged to the Plaintiff. The suit was not successful.
(v) Rs 10,000 were spent on advertising the introduction of a new product in the market,
the benefit of which will be effective during four years.
(vi) A factory shed was constructed at a cost of Rs 1,00,000. A sum of Rs 5,000 had been
incurred for the construction of the temporary huts for storing building materials.
Answer 1.
(i) Rs. 10,000 incurred in connection with obtaining a license for starting the factory is a Capital
Expenditure. It is incurred for acquiring a right to carry on business for a long period.
(ii) Rs. 1,000 incurred for removal of stock to a new site is treated as a Revenue Expenditure
because it is not enhancing the value of the asset and it is also required for starting the
business on the new site.
(iii) Rs. 5,000 incurred for changing Rings and Pistons of an engine is a Revenue Expenditure
because, the change of rings and piston will restore the efficiency of the engine only and it
will not add anything to the capacity of the engine.
(iv) Rs. 2,000 incurred for defending the title to the firm¡¯s assets is a Revenue Expenditure.
(v) Rs. 10,000 incurred on advertising is to be treated as a Deferred Revenue Expenditure because
the benefit of advertisement is available for 4 years, Rs. 2,500 is to be written off every year.
(vi) Cost of construction of Factory shed of Rs. 1,00,000 is a Capital Expenditure, similarly cost
of construction of small huts for storing building materials is also a Capital Expenditure.
Q2. The financial year of Mr. Chalaman ends on 31st March, 2008 but the stock in hand was
physically verified only on 8th April, 2008. You are required to determine the value of
Closing Stock (at cost) as at 31st March, 2008 from the following information.
(i) The stock (valued at cost) as verified on 8th April, 2008 was Rs. 15,000.
(ii) Sales have been entered in the Sales Day Book only after the despatch of goods and sales
returns only on receipt of goods.
2 Revisionary Test Paper (Revised Syllabus-2008)
(iii) Purchases have been entered in the Purchase Day Book on receipt of the purchase invoice
irrespective of the date of receipt of the goods.
(iv) Sales as per the sales day book for the period 1st April, 2008 to 8th April, 2008 (before the
actual verification) amounted to Rs. 6,000 of which goods of a sale value of Rs. 1,000 had
not been delivered at the time of verification.
(v) Purchases as per the purchase day book for the period 1st April, 2008 to 8th April, 2008
(before the actual verification) amounted to Rs. 6,000 of which purchases valued Rs. 1,500
had not been received at the date of verification and purchases of Rs. 2,000 had been received
prior to 31st March, 2008.
(vi) In respect of goods costing Rs. 5.000 received prior to 31st March, 2008, invoices had not
been received up to the date of verification of stocks.
(vii) The gross profit is 20% on sales.
Solution:
Mr. Chalaman
Statement showing Value of Stock on 31.3.2008
Particulars Amount Amount
Rs. Rs.
Stock as on 8.4.08 15,000
Add :
(a) Cost of Goods Sold and sent Out between
1.4.08 and 8.4.08
Sales in this period 6,000
Less : Goods sold but not delivered (at Selling Price) 1,000
5,000
Less : Gross Profit included [20% of 5,000] 1,000 4,000
19,000
Less :
(a) Goods purchased and received between
1.4.08 and 8.4.08 :
Purchases in this period 6,000
Less : Goods not received till 8.4.08 1,500 4,500
(b) Goods received before 31 .3.08 for which
the invoice is yet to be received 5,000
Stock on 31.3.2008 9,500
Q3. (a) State the essential features of Accounting Principles.
(b) Explain the Convention of Materiality.
(c) State the situations where the principle of conservatism is applicable.
Answer:
(a) Essential Features of Accounting Principles
Accounting principles are acceptable when they, in general, satisfy the following three
basic norms
Group-I : Paper-5 : Financial Accounting 3
(i) usefulness,
(ii) objectivity, and
(iii) feasibility.
Accounting principles satisfy the first basic feature of usefulness as much as because of
these, the accounting records become more meaningful and useful to the reader. In other
words, an accounting rule, which does not increase the utility of the records to its readers,
is not accepted as an accounting principle.
Accounting principles is that which is objective in nature. It is said to be objective when it is
solidly supported by facts. It is objective when it cannot be influenced by the personal bias
and whims.
Accounting principles should be such as are practicable. It will be seen that assets in the
accounts are recorded at cost less depreciation as against at market price. This accounting
principle is practicable and feasible because it does not entail the difficult work of ascertaining
the market price of that asset. It does not make it obligatory for the accountant to record
all fluctuation in the price of that asset.
(b) Materiality is primarily related to the qualitative characteristic ¡®relevance¡¯. If an item is not
material, then it is not relevant. Para 17 of AS 1 states that financial statements should
disclose all material items. Material items are those the knowledge of which might influence
the decisions of the user of the financial statements. Materiality depends on the size of item
or error judged in the particular circumstances of its omission or misstatement. From a
positive perspective, materiality has to do with the significance of an the item or event to
warrant attention in the accounting process. From a negative view point, materiality is
critical because otherwise a great deal of time might be spent on trivial matters in the
accounting process. Materiality provides a threshold or cut-off point rather than being a
primary qualitative characteristic which information must have if it is to be useful. The
Financial Accounting Standards Board, USA expresses the opinion that ¡°no general standards
of materiality can be formulated to take into account all the considerations that enter into
an experienced human judgement¡±. Individual judgements are required to assess materiality,
or to decide what the appropriate minimum quantitative criteria are to be set for given
situations. The Companies Act, 1956 also recognises the need for separate disclosure of
material items. Part II of Schedule VI states that any item of expense which exceeds 1% of
the total revenue of the company or Rs. 5,000, whichever is higher, should be shown as a
separate and distinct item against an appropriate head in the profit and loss account.
(c) ¡®Conservatism convention¡¯ states that the accountants should not anticipate income and
should provide for all possible losses. The underlying principle is that revenues should only be
recognised when there is reasonable certainty about their realisation. At the same time
provision must be made for all possible liabilities, whether the amount is known with certainty
or is based on estimates. Faced with the choice between two methods of valuing an asset,
the method which leads to lesser value must be selected. To illustrate, inventories are
recorded at the cost or market value, whichever is less or if there is a possibility that a debt
may not be realised, a specific amount is set aside from profits as a provision for doubtful
debts.
4 Revisionary Test Paper (Revised Syllabus-2008)
Q4. On 1.4.2003 Mayami got a mining lease and from that date a part of the mine was subleased
to Pathan. The terms of payment and the production of 5 years are as below.
Particulars Lessee Sub-Lessee
Royalty (Rs/Tonne) 2.00 3.00
Dead Rent per anum(Rs) 15,000 10,000
Short working recoverable(Years) 3 2
Production (Year ended....31.3.)
2004 1,000 1,000
2005 3,000 2,000
2006 12,000 5,000
2007 9,000 2,000( strike)
2008 5,000 12,000
In case of strike, royalty earned will discharge all liabilities for the year only.
Show ledger accounts in the books of Mayami.
Solution :
In the Books of Mayami
Statement showing Royalties Payable
Year Output Actual Min. Excess Short Workings Amount
Ended Written
31.3 (Tons) Royalties Rent Workings Occurred Recouped off C/F Payable
Rs. Rs. Rs. Rs. Rs. Rs. Rs. Rs.
2004 2,000 4,000 15,000 0 11,000 0 0 11,000 15,000
2005 5,000 10,000 15,000 0 5,000 0 0 16,000 15,000
2006 17,000 34,000 15,000 19,000 0 16,000 0 0 18,000
2007 11,000 22,000 15,000 7,000 0 0 0 0 22,000
2008 17,000 34,000 15,000 19,000 0 0 0 0 34,000
Statement Showing Royalties Receivable
Year Output Actual Min. Excess Short Workings Amount
Ended Written
31.3 (Tons) Royalties Rent Workings Occurred Recouped off C/F Payable
Rs. Rs. Rs. Rs. Rs. Rs. Rs. Rs.
2004 1,000 3,000 10,000 0 7,000 0 0 7,000 10,000
2005 2,000 6,000 10,000 0 4,000 0 0 0 11,000
2006 5,000 15,000 10,000 5,000 0 5,000 2,000 0 4,000
2007 2,000 6,000 6,000 0 0 0 4,000 0 -
2008 12,000 36,000 10,000 26,000 0 0 0 0 36,000
Group-I : Paper-5 : Financial Accounting 5
Dr. Short working Suspense Account Cr.
Date Particulars Amt (Rs) Date Particulars Amt (Rs)
31.03.06 To Balance c/d 7,000 31.03.02 By Pathan A/c 7,000
7,000 7,000
31.03.03 To Balance c/d 11,000 1.04.02 By Balance b/d 7,000
31.03.03 By Pathan A/c 4,000
31.3.06 To Pathan A/c 5,000 1.4.05 By Balance b/d 11,000
To P& L A/c 2,000
To Balance c/d 4,000
11,000 11,000
31.03.07 To P &L A/c 4,000 1.4.06 By Balance b/d 4,000
4,000 4,000
Dr ROYALTY RECEIVABLE ACCOUNT Cr
Date Particulars Amt (Rs) Date Particulars Amt (Rs)
31.03.04 To Royalties Payable A/c 2000 31.03.04 By Pathan A/c 3,000
(1000*2) (1000*3)
To Profit & Loss A/c 1,000
3,000 3,000
31.03.05 To Royalties Payable A/c 4000 31.03.05 By Pathan A/c 6,000
(2000*2) (2000*3)
To Profit & Loss A/c 2,000
6,000 6,000
31.03.06 To Royalties Payable A/c 10000 31.03.06 By Pathan A/c 15,000
(5000*2) (5000*3)
To Profit & Loss A/c 5,000
15,000 15,000
31.03.07 To Royalties Payable A/c 4000 31.03.07 By Pathan A/c 6,000
(2000*2) (2000*3)
To Profit & Loss A/c 2,000
6,000 6,000
31.03.08 To Royalties Payable A/c 24000 31.03.08 By Pathan A/c 36,000
(12000*2) (12000*3)
To Profit & Loss A/c 12,000
36,000 36,000
6 Revisionary Test Paper (Revised Syllabus-2008)
Dr ROYALTY PAYABLE ACCOUNT Cr
Date Particulars Amt (Rs) Date Particulars Amt (Rs)
31.03.04 To Landlord A/c 4,000 31.03.04 By R/R A/c 2,000
By P & L A/c 2,000
4,000 4,000
31.3.05 To Landlord A/c 10,000 31.3.05 By R/R A/c 4,000
By P/L A/c 6,000
10,000 10,000
31.3.06 To Landlord A/c 34,000 31.3.06 By R/R A/c 10,000
By P/L A/c 24,000
34,000 34,000
31.3.07 To Landlord A/c 22,000 31.3.07 By R/R A/c 4,000
By P/L A/c 18,000
22,000 22,000
31.3.08 To Landlord A/c 34,000 31.3.08 By R/R A/c 24,000
By P/L A/c 10,000
34,000 34,000
Pathan¡¯s A/c
Date Particulars Amount Date Particulars Amount
(Rs) (Rs)
31.03.04 To R/R A/c 3,000 31.03.04 By Bank A/c 10,000
To S/W Suspense A/c 7,000
10,000 10,000
31.03.05 To R/R A/c 6,000 31.03.04 By Bank A/c 10,000
To S/W Suspense A/c 4,000
10,000 10,000
31.03.06 To R/R A/c 15,000 31.03.04 By Bank A/c 15,000
15,000 15,000
31.03.07 To R/R A/c 6,000 31.03.04 By Bank A/c 6,000
6,000 6,000
31.03.08 To R/R A/c 36,000 31.03.04 By Bank A/c 36,000
36,000 36,000
Group-I : Paper-5 : Financial Accounting 7
Land Lord¡¯s A/c
Date Particulars Amount Date Particulars Amount
(Rs) (Rs)
31.03.04 To Bank A/c 15,000 31.03.04 By Royalty
Payable A/c 4,000
By S/W A/c 11,000
15,000 15,000
31.03.05 To Bank A/c 15,000 31.03.05 By Royalty
Payable A/c 10,000
By S/W A/c 5,000
15,000 15,000
31.03.06 To Bank A/c 18,000 31.03.04 By Royalty
To S/W A/c 16,000 Payable A/c 34,000
34,000 34,000
31.03.07 To Bank A/c 22,000 31.03.07 By Royalty
Payable A/c 22,000
31.03.08 To Bank A/c 34,000 31.03.08 By Royalty 34,000
Payable A/c
Q5. The hire purchases depaetment of Zapak Ltd provides you the following information for the
year ending on 30th September 2008 :
Purchase cost per unit Rs 3,000
Cash sales price per unit Rs 4,000
Cash down payment per unit Rs 400
Monthly payment per unit Rs 350
Number of instalments per unit 12
Number of units sold on hire purchase basis 120
Number of instalments collected 420
Nuber of instalments due but not yet collected 58
Required : Calculate the following :
(a) Number of instalments fallen due during the year, (b) Number of instalments not yet due on
30. 9. 2008, (c) Amount of instalment not yet due, (d) Amount of istalment due but not yet
collected, (e) Amount fallen due during the year, (f) Cash collected during the year, (g) Hire Purchase
price per unit, (h) Total hire purchase price of units sold, (i) Total cost price of units sold on hire
purchase, (j) % of profit margin on H.P. Sales, (k) Profit included in total hire purchases price, (l)
Profit included in the amount of instalments not yet due, (m) Gross Profit.
Also Prepare Hire Purchase Stock Account, Hire Purchases Debtors Account and Hire Purchase
Adjustment Acount.
8 Revisionary Test Paper (Revised Syllabus-2008)
Solution :
Step 1: Prepare the Analysis of No. of Instalments
Total No. of Instalments = No. of Units sold x Total No. of Instalments Payable
= 120 x 12 = 1440
Again, Total No. of instalments = Instalment Due + Instalments not Due
Where, Instalments Due= Instalments Received + Instalments not received= 420 + 58 =478
Instalments Not Due = Total No of Instalments . Instalments Due
= 1440 . 478 = 962 (Balancing Figure)
Step 2 : Prepare the Analysis of Total Hire Purchase Price
Total Hire Purchase Price= No. of Units sold x H.P. Selling price per unit
= 120 x 4,600 = Rs 5,52,000
Again, Total Hire Purchase Price=Down Payment received+ Instalments Receivable
Down Payment Received = No. of units sold x Down Payment per unit
= 120 x Rs 400 = Rs 48,000
Instalments Receivable = 1,440 x Rs.350= Rs.5,04,000
Again, Instalments Receivable=Instalments Due +Instalments not Due
Where, Instalments Due= 478 x Rs 350 = Rs 1,67,300
Instalments Not Due = 962 x Rs 350 = Rs.3,36,700
Again, Instalments Due= Instalments due & received + Instalments due but not received
Where, Instalments due & received = 420 x 350 = 1,47,000
Instalments due but not received = 58 x Rs 350 = Rs 20,300
Let us answer the given questions on the basis of above :
(a) Number of Instalments fallen due = 478
(b) Number of Instalments not yet due = 962
(c) Amount of Instalments not yet due = Rs 3,36,700
(d) Amount of Instalments due but not yet collected = Rs 20,300
(e) Total Amount fallen due = Down Payment + Instalments due
= Rs 48,000 + Rs 1,67,300 = Rs 2,15,300
(f) Total Cash Collected = Down Payment + Instalments due and received
= Rs 48,000 + Rs 1,47,000 = Rs 1,95,000
(g) Hire Purchase Price per unit = Down Payment/Unit + Amount of Instalment/Unit
= Rs 400 + (Rs 350 x 12) = Rs 4,600
Group-I : Paper-5 : Financial Accounting 9
(h) Total Hire Purchase Price = Rs 5,52,000
(i) Total Cost Price of Units sold on H.P. = No. of Units sold Cost per unit
= 120 x Rs 3,000 = Rs 3,60,000
(j) % of Profit Margin on H.P. Sales = [{(H. P. Price - Cost Price) / H. P. Price} x (100)]
= [{(4,600 - 3,000) / 4,600} x (100)]
= 34.78%
(k) Profit included in Total Hire Purchase P = [{(4,600 - 3,000) / 4,600} x (5,52,000)]
(l) Profit included in the Total Amount of Instalments not yet due
= [{(4,600 - 3,000) / 4,600} x (3,36,700)]
= 1,17,113
(m) Gross Profit on Hire Purchase Sales = % of Profit Margin x Total Amount fallen due
= [{(4,600 - 3,000) / 4,600} x (2,15,300)]
= 74,887
Dr. (i) Hire Purchase Stock Account Cr.
Particulars Rs Particulars Rs
To Balance b/d . By Hire Purchase Debtors A/c 2,15,300
To Goods Sold on Hire Purchase A/c 5,52,000 By Balance c/d 3,36,700
5,52,000 5,52,000
Dr. (ii) Hire Purchase Debtors Account Cr.
Particulars Rs Particulars Rs
To Balance b/d . By Bank A/c 1,95,000
To Hire Purchase Stock A/c 2,15,300 By Balance c/d 20,300
2,15,300 2,15,300
Dr. (iii) Hire Purchase Adjustment Account Cr.
Particulars Rs Particulars Rs
To Hire Purchase Stock Reserve A/c By Goods Sold on Hire
[3,36,700 ¡¿1,600/4,600] 1,17,113 Purchase A/c 1,92,000
To Profit t/f to Profit & Loss A/c 74,887 [5,52,,000¡¿1,600/4,600]
1,92,000 1,92,000
10 Revisionary Test Paper (Revised Syllabus-2008)
Q6. Mr. Gulab sells goods on hire purchase basis. He fixes hire purchase price at 331/3%profit on
invoice price of the goods. The following are the figures relating to his hire purchase business
for the year ending on 31st March 2008 :
01.04.2007 31.03.2008
Rs. Rs.
Hire Purchase Stock 60,000 ?
Hire Purchase Debtors 1,500 ?
Shop Stock 50,000 75,000
Goods purchased during the year Rs 3,27,000, Cash received from customers during the year Rs
4,62,000. Total amount of instalments that fell due during the year Rs 4,63,500.
One customer to whom goods had been sold for Rs 6,000 paid only 5 instalments of Rs 500 each.
On his failure to pay the monthly instalment of Rs 500 each on 4th March 2008, the goods were
repossessed on 27th March 2008 after due legal notice.
Required : Prepare the Hire Purchase Trading Account.
Solution :
Dr. Hire Purchase Trading Account Cr.
Particulars Rs Particulars Rs
To Opening Balances: By Hire Purchase Stock Reserve
Hire Purchase Stock 60,000 [60,000 ¡¿ 50/150] 20,000
Hire Purchase Debtors 1,500 By Bank A/c 4,62,000
To Goods Sold on Hire Purchase 4,53,000 By Goods Sold on Hire
To Hire Purchase Stock Reserve A/c 15,500 Purchase A/c 1,51,000
[46,500¡¿50/150] [ 4,53,000 x 50/150]
To Profit t/f to General By Goods Repossessed A/c 2,333
P & L A/c 1,54,333 [At Revalued Figure]
By Closing Balances :
Hire Purchase Stock 46,500
Hire Purchase Debtors 2,500
6,84,333 6,84,333
Working Notes :
Dr. (i) Shop Stock Account Cr.
Particulars Rs Particulars Rs
To Balance b/d 50,000 By Goods Sold on Hire Purchase A/c 3,02,000
To Purchases 3,27,000 By Balance c/d 75,000
[Excluding Goods Repossessed]
3,77,000 3,77,000
Group-I : Paper-5 : Financial Accounting 11
Dr. (ii) Goods Sold on Hire Purchase Account Cr.
Particulars Rs Particulars Rs
To Stop Stock A/c 3,02,000 By Hire Purchase Trading A/c 4,53,000
To Hire Purchase Trading A/c 1,51,000
4,53,000 4,53,000
Dr. (iii) Memorandum Hire Purchase Stock Account Cr.
Particulars Rs Particulars Rs
To Balance b/d 60,000 By Hire Purchase Debtors A/c 4,63,500
To Goods Sold on Hire Purchase 4,53,000 By Goods Repossessed A/c 3,000
By Balance c/d 46,500
4,65,000 4,65,000
Dr. (iv) Memorandum Hire Purchase Debtors Account Cr.
Particulars Rs Particulars Rs
To Balance b/d 1,500 By Bank A/c 4,62,000
To Hire Purchase Stock A/c 4,63,500 By Goods Repossessed A/c 500
By Balance c/d 2,500
4,65,000 4,65,000
Working Note :
Calculation of the value of Goods Repossessed.
= (Cost Price / Hire Purchase Price) ¡¿ Unpaid amount ( whether due or not)
=


¡¿ ÿ
.
.
..
.
= Rs. 2,333
Q7. The Dreamers¡¯ Club makes up its accounts to 31st December in each year. On 31st December,
2008 the cashier of the club absconded leaving behind no information or cash. An examination
of the records showed that the books had not been written up for a considerable time and it
was decided to reconstruct the figures from 1.1.2008.
A summary of the Bank Account for the year showed that :
Receipts Amount Payments Amount
Rs. Rs.
Balance on 1.1.2008 420 Rent & Rates 460
Bank Deposits 42,610 Insurance 40
Light & Heat 156
Bar Purchases 35,067
Telephone 59
Cash Withdrawn 5,848
Balance as on31.12.08 1,400
43,030 43,030
12 Revisionary Test Paper (Revised Syllabus-2008)
The following information is also obtained :
I. The Barman places takings in the bank night safe on his way home for crediting to the club
account. The bartakings totalled Rs. 44,610 for the year. The treasurer had no access to bar
takings.
2. The receipt counterfoils for members subscriptions totaled Rs. 3,050 for the year.
3. A summary of expenditure for petty cash and wages revealed Glasses, crockery main
tenance. Rs. 1,310; Wages.Rs. 2,650; Sundry Expenses.Rs. 475
4. Outstanding or Prepaid Amounts were :
31.12.07 31.12.08
Prepaid Rates Rs. 26 Rs. 28
Outstanding Expenses Rs. 64 Rs. 100
The Bar Stock on 1.1.2008 was Rs. 3,607 and 31.12.08 Rs. 2,916. Opening Cash with the
Cashier at the beginning of the year 2008 was Rs. 35 only.
Prepare an Income & Expenditure Account of the club for the year ended 31.12.2008.
Working Notes:
1. Cash Defalcation by Cashier :
Cash Account
Particulars Amount Particulars Amount
Rs. Rs.
To Balance b/f 35 By Bank (44,610 . 42,610) 2,000
To Subscriptions 3,050 By Glasses, crockery etc. 1,310
To Bank 5,848 By Wages 2,650
By Sundry Expenses 475
By Defaltation of cash 2,498
8,933 8,933
2. Rent & Rates
Paid from Bank4 60
+ Prepaid as on 31.12.07 (relating to current year) 26
486
. Prepaid Rates as on 31.12.08(relating to next year) 28
458
Group-I : Paper-5 : Financial Accounting 13
2. Bar Trading A/c for the year ended 31.12.08
Particulars Amount Particulars Amount
Rs. Rs.
To Opening Stock 3,607 By Bar takings 44,610
To Purchases 35,067 By Closing Stock2,916
To Income & Expenditure A/c 8,852
47,526 47,526
Dreamers¡¯ Club
Dr. Income and Expenditure Account for the year ended 31.12.2008 Cr.
Particulars Amount Particulars Amount
Rs. Rs.
To Rent & Rates 458 By Subscriptions 3,050
To Insurance 40 By Bar Trading A/c 8.852
(Profit from Bar)
To Light & Heat 156
To Telephone Charges 59
To Glass & Crockery Maintenance 1,310
To Wages 2,650
To Expenses 475
Add : Outstanding for 08 100
Less : Outstanding for 07 64 511
To Defalcation of Cash 2,498
To Surplus (Excess of Income
over expenditure) 4,220
11,920 11,920
Q8. The Accountant of City Club furnished the following information about the Receipts and
Payments of the club for the year ended 31st March, 2009 :
Receipts Rs. Payments Rs.
To Subscriptions 62,130 By Premises 30,000
¡° Fair Receipts 7,200 ¡° Rent 2,400
¡° Variety show Receipts (net) 12,810 ¡° Rates and Stationery 3,780
¡° Interest 690 ¡° Printing & Stationery 1,410
¡° Bar Collections 22,350 ¡° Sundry Expenses 5,350
¡° Wages 2,520
¡° Fair Expenses 7,170
¡° Honorarium to Secretary 11,000
¡° Bar Purchases (Payment) 17,310
¡° Repairs 960
¡° New Car (less proceeds of
old car Rs.9,000) 37,800
14 Revisionary Test Paper (Revised Syllabus-2008)
The following additional information could be obtained :
1.4.08 1.3.09
(Rs.) (Rs.)
Cash in hand 450 NIL
Bank Balance as per Cash Book 24,420 10,350
Cheque issued for Sundry Expenses not presented 270 90
to the bank (entry has been duly made in the Cash
book)
Subscriptions Due 3,600 2,940
Premises at cost 87,000 117,000
Provision for Depreciation on Premises 56,400
Car at cost 36,570 46,800
Accumulated Depreciation on Car 30,870
Bar Stock 2,130 2,610
Creditors for Bar Purchases 1,770 1,290
Annual Honorarium to Secretary is Rs. 12,000 Depreciation on Premises is to be provided at 5%
on written down value. Depreciation on new car is to be provided at 20%.
You are required to prepare Receipts and Payments Account and Income and Expenditure Account
for the year ended 31.3.09.
Solution :
Working Notes :
Rs.
(1) Depreciation on New Car :
Net Amount 37,800
Add : Sale proceeds of Old Car 9,000
Actual Cost 46,800
Less : Depreciation @ 20% 9,360
37,440
(2) Profit on sale of Old Car :
Sale proceeds 9000
Less: Written Down Value : Cost - 36,570
Provision for Depreciation - 30,870 5700
Profit on Sale 3300
(3) Cheques issued for Sundry Expenses not presented to the Bank need not be considered as
Bank Balance as per Cash Book is given and the entry for the expenses have been duly made
in the Cash Book.
Group-I : Paper-5 : Financial Accounting 15
(4) Calculation of Bar Parchases
Dr. Creditors for Bar Purchases Account Cr.
Particulars Rs. Particulars Rs.
31.3.09 To cash Payment for 1.4.08 By Balance b/d 1,770
Bar Purchases 17,310
31.3.09 To Balance c/d 1,290 31.3.09 By Purchases
(Balance Figure) 16,830
18,600 18,600
(5)
Dr. Bar Trading Account for the year ended 31.03.09 Cr.
Particulars Rs. Particulars Rs.
To Opening Stock 2,130 By Bar collections 22,350
To Bar Purchases 16,830 By Close stock2,610
To Income & Expenditure A/c
profit from Bar transfered 6,000
24,960 24,960
City Club
Receipts & Payments Account for the year ended 31.3.09
Receipts Rs. Payments Rs.
To Balance b/d : By Premises 30,000
¡° Cash in hand 450 ¡° Rent 2,400
¡° Cash at Bank 24,420 ¡° Rates & Taxes 3,780
¡° Subscriptions 62,130 ¡° Printing & Stationery 1,410
¡° Sundry Expenses 5,350
¡° Fair Receipts 7,200 ¡° Wages 2,520
¡° Variety Show Receipts (Net) 12,810 ¡° Fair Expenses 7,170
¡° Interest 690 ¡° Honorarium to Secretary 11,000
¡° Bar Collections 22,350 ¡° Payments for Bar Purchases 17,310
¡° Sale Proceeds of Old Car 9,000 ¡° Repairs 960
¡° Cost of New Car 46,800
¡° Balance c/d : Cash at Bank 10,350
1,39,050 1,39,050
16 Revisionary Test Paper (Revised Syllabus-2008)
City Club
Dr. Income and Expenditure Account for the year ended 31st March, 2009 Cr.
Expenditure Rs. Rs. Income Rs. Rs.
To Rent 2,400 By Subscription 62,130
Rates & Taxes 3,780 Add : Amount Due
On 31.3.09 2,940
¡° Printing & Stationery 1,410 65,070
¡° Wages 2,520 Less: Amount Due
¡° Honorarium to Secy. 11000 On 31.3.08 3,600 61470
¡° Add: O/S on 31.3.09 1000 12,000 ¡° Profit on Sale of
¡° Sundry Expenses 5,350 Old Car [Note 1] 3300
¡° Repairs 960 ¡° Profit from Bar
¡° Depreciation : [Note 5] 6000
On Car [Note 1] 9360 ¡° Variety Show
On Premises
[5% of 60600] 3030 12,390 Receipts (net) 12810
¡° Income from Fair :
¡° Surplus (Excess of Receipts 7200
Incomes over Less : Expenses 7170 30
Expenditure, transfer 43,490 ¡° Interest 690
to Capital Fund)
84,300 84,300
Baisakhi and Srabarni are partners sharing profits and losses in proportion to their capitals. Their
Balance Sheet as on 31st March, 2009 is given below :
Liabilities Rs. Assets Rs.
Creditors 15,000 Freehold Premises 10,000
General Reserve 2,100 Machinery 3,500
Capitals : Furniture 1.750
Baisakhi 20,000 Office Equipments 550
Srabani 15,000 Stock 14,100
Bill Receivable 3,060
Debtors 17,500
Bank 1,590
Cash 50
52,100 52,100
Group-I : Paper-5 : Financial Accounting 17
On 1st April, 2009 they admit Poushali on the following conditions:
(i) Poushali should bring in Rs. 10,000 as capital and to pay Rs. 3,500 for goodwill as she will
get 1/4th share in profits.
(ii) A provision of 2% to be raised against debtors, stock to be reduced by 5%, Freehold
Premises to be revalued at Rs. 12,650, Machinery at Rs. 2,800, Furniture at Rs. 1,540
and Office equipments at Rs. 495.
(iii) Partners agreed that the values of assets and liabilities should remain unaltered.
Show the necessary accounts and prepare the opening Balance Sheet of the new firm.
Points to be noted :
1. The Partners have decided not to alter the book values of the assets and liabilities. The
effects of revaluation may be ascertained either by preparing a Memorandum Revaluation
Account as follows.
(a) Calculation of Profit/Loss on Revaluation
Dr. Memorandum Revaluation Account Cr.
Rs. Rs.
To PBDD 350 By Freehold Premises 2,650
(@ 2% of 17,500)
ToStock7 05
To Machinery 700
To Funiture 210
To Office Equipments 55
To Partners Capital A/c¡¯s
Baisakhi : (4/7) 360
Sarbani : (3/7) 270
2,650 2,650
To Reversal of Items b/d 2650 By Reversal of Items b/d 2,020
By Partners Capital A/c
(In New Ratio)
[Loss on Revaluation]
Baisakhi 270
Sharbani 203
Poushali 157 630
2,650 2,650
(b) As General Reserve is to remain unaltered, similar adjustment will be required to be shared
among old partners in old ratio and then written back among all partner¡¯s in new ratio
18 Revisionary Test Paper (Revised Syllabus-2008)
2. Calculation of net effects on Capital Accounts
New Profit Sharing Ratio : 12 : 9 : 7
Dr. Capital Accounts Cr.
Baisakhi Srabani Poushli Baisakhi Srabani Poushali
Date Particulars Amount Amount Amount Date Particulars Amount Amount Amount
Rs. Rs. Rs. Rs. Rs. Rs.
31.3.08 To Gen. Res. 900 675 525 1.4.07 By Balance b/d 20000 15000
To M. Rev. A/c 270 203 157 By Gen. Reserve 1,200 900 .
31.3.08 By Bank A/c . . 10000
By M.Rev. A/c 360 270 .
By Bank A/c 2000 1500 .
(Premium) at 4:3
¡° Balance c/d 22,390 16,792 9,318
23,560 17,670 10,000 23,560 17,670 10,000
Balance Sheet as on 1.4.2008
Liabilities Rs. Assets Rs.
Capitals : Freehold Premises 10,000
Baisakhi 22,390 Machinery 3,500
Srabani 16,792 Furniture 1,750
Poushali 9,318 48,500
General Reserve 2,100 Office Equipment 550
Sundry Creditors 15,000 Stock14,100
Bills Receivable 3,060
Debtors 17,500
Bank
[1,590 +10,000+3,500] 15,090
Cash 50
65,600 65,600
Q10. Kalyani and Ranu commenced business on 1st July, 2006 as partners with capitals of Rs.
1,80,000 and Rs. 1,20,000 respectively. The capitals would remain fixed and carry interest
at 10% p.a. profit and losses were to be shared in proportion to their capitals.
They appointed Anita as their Manager on 1st July, 2006 at a salary of Rs. 9,600 per
annum plus a bonus of 5% of the net profits after charging such bonus and interest as a
partner from the commencement of the business. She had to deposit Rs. 80,000 as security,
carrying an interest @ 12%p.a. It was agreed that she would be entitled to one-fifth share
of the profits and her security deposit would be treated as her capital carrying interest @
10% p.a. It was further agreed that this new arrangement should not result in Anita¡¯s share
for any of these years being less than what she had already received under the original
agreement and terms of her appointment.
Group-I : Paper-5 : Financial Accounting 19
The profits before charging Anita¡¯s bonus and interest on Capital of the partners or giving effect
to the new arrangement were . (a) for the year 2006-07 - Rs. 60,000; (b) for the year 2007 . 08
. Rs. 1,20,000; (c) for the year 2008-09 . Rs. 1,60,000
Show by a single journal entry to give effect to the new arrangement with explanatory computation.
Points to be noted :
1. As a Manager, Anita received (a) bonus @ 5% on Net Profits after charging such bonus and
interest on capital at 10% p.a. to Kalyani and Ranu (b) Salary Rs. 9,600 p.a. (c) Interest on
security deposit at 12% p.a.
2. As a Partner Anita is entitled to (a) Interest on Capital at 10% p.a. (b) 1/5th of profit after
providing interest on capital at 10% p.a. to all partners including herself.
3. If total dues of Anita under (2) above is more than that under (1) above, she should get the
difference. But if such dues under (1) above is more, she would not refund the excess already
received.
Solution :
(1) Workings : Calculation of Anita¡¯s Dues as Manager
2006-07 2007-08 2008-09
Rs. Rs. Rs.
Salary 9,600 9,600 9,600
Interest on Security Deposit : 12% of 80,000 9,600 9,600 9,600
Bonus 5/105 of profit after charging interest on capitals
of Kalyani and Ranu
2006-07 = 5/105 of (60,000 - 10% of 3,00,000) 1,429
2007-08 = 5/105 of 1,20,000 . 10% of 3,00,000) 4,286
2008-09 = 5/105 of 1,60,000 . 10% of 3,00,000) 6,190
20,629 23,486 25,390
(2) Calculation of Distributable profit under the new arrangement
2006-07 2007-08 2008-09
Rs. Rs. Rs.
Net profits given (after charging interest on security deposit
and Anita¡¯s salary but before charging interest on capitals) 60,000 1,20,000 1,60,000
Add : Anita¡¯s Salary and Interest on Deposit no more
payable [9,600 +9,600] 19,200 19,200 19,200
79,200 1,39,200 1,79,200
Less : Interest on Capitals to all partners @ 10% of 38,000 38,000 38,000
[1,80,000 + 1,20,000 + 80,000]
Distributable Profits 41,200 1,01,200 1,41,200
Anita¡¯s Share of Profit = 1/5th of Distributable Profit 8,240 20,240 28,240
20 Revisionary Test Paper (Revised Syllabus-2008)
(3) . Difference in Payments to Anita
2006-07 2007-08 2008-09
Rs. Rs. Rs.
A. Anita¡¯s Dues as Partner :
Interest on Capital @ 10% of 80,000 8,000 8,000 8,000
Share of Profit [as perworkings 2] 8,240 20,240 28,240
16,240 28,240 36,240
B. Anita¡¯s Dues as manager [as perworkings 1] 20,629 23,486 25,390
Difference Payable to Anita . 4,754 10,850
Total 15,604
Journal
Dr. Cr.
Date Particulars L.F Amount Amount
Rs. Rs.
Kalyani¡¯s Current A/c [3/5 of 15,604] Dr. 9,362
Ranu¡¯s Current A/c [2/5 of 15,604] Dr. 6,242
To Anita¡¯s Current A/c
[Adjustments made through Partners¡¯ Current 15,604
A/cs to the to new arrangement regarding profits]
Q11. The following was the Balance Sheet of A, B and C who shared profits in the ratio of
1 : 2 : 2 as on 31st December, 2008.
Sundry Creditors 10,000 Goodwill 15,000
Capital A/c: Debtors 10,000
A 10,000 Machinery 20,000
B 20,000 Buildings 30,000
C 20,000 50,000 Stock10,000
General Reserve 5,000 Cash at Bank 5,000
Investment Fluctuation Fund 3,000 Investments 10,000
Bad Debts Reserve 2,000
Bank Loan 30,000
1,00,000 1,00,000
C died on 31st March, 2009. His account is to be settled under the following terms :
Goodwill is to be calculated at the rate of 2 years purchase on the basis of the average of 5 years
profit or loss. Profit for January to March¡¯ 09 is to be calculated proportionately on the average
profit of 3 years. The profits were : 2004 Rs. 3,000, 2005 Rs. 7,000, 2006 Rs. 10,000, 2007
Rs. 14,000, 2008 loss Rs. 12,000. During 2008 a Moped costing Rs. 4,000 was purchased and
Group-I : Paper-5 : Financial Accounting 21
debited to Travelling Expenses Account on which depreciation is to be calculated @ 25%/. Other
values agreed on assets are : Stock Rs. 12,000, Building Rs, 35,000, Machinery Rs,. 25,000 and
Investments Rs. 8,000. Debtors are considered good.
Prepare new Balance Sheet of the firm, necessary Journal entries and Ledger Accounts of the
Partners.
Solution :
1. Adjusted profit for 2008
Profit (12,000)
Add : Cost of Moped
Wrongly treated as Travelling Expense 4,000
Less : Depreciation not charged on Moped @25%
on Rs. 4,000 (1,000)
Adjusted Profit (9,000)
2. Valuation of Goodwill
Total Profit/Loss for the last 5 years= 3,000+7,000+10,000+14,000 . 9,000=
Rs. 25,000
Average Profit = Rs. 25,000/5 = Rs. 5,000; Goodwill = 2¡¿ Rs. 5,000 = Rs. 10,000
But Goodwill is appearing at Balance Sheet at Rs. 15,000. Over valuation of Goodwill Rs. 5,000
should be written off among A, B & C as 1 : 2 : 2.The balance of Goodwill between A & B in
the ratio 1:2
3. Share of Profit of Deceased Partner till his date of death
Average Profit of the last 3 years [ 2006, 2007 & 2008] = (10,000 + 14,000 . 9,000)/3
= Rs. 5,000
Estimated Profit for 3 months [Jan to March, ¡®09] = Rs. 5,000¡¿3/12 = Rs. 1,250
C¡¯s share of profit = Rs. 1,250¡¿2/5 = Rs. 500
Solution :
In the Books of A, B & C
Journal Entries
Date Particulars Amount Amount
Rs. Rs.
Stock A/c Dr. 2,000
Buildings A/c Dr. 5,000
Machinery A/c Dr. 5,000
Moped A/c [4,000 .Depr. 1,000] Dr. 3,000
To Revaluation A/c 15,000
[Values of assets increased on revaluation]
22 Revisionary Test Paper (Revised Syllabus-2008)
Date Particulars Amount Amount
Rs. Rs.
General Reserve A/c Dr. 5,000
Investment Fluctuation Fund A/c Dr. 3,000
Bad Debts Reserve A/c Dr. 2,000
To A¡¯s Capital A/c 2,000
To B¡¯s Capital A/c 4,000
To C¡¯s Capital A/c 4,000
[Transfer of Reserves etc. to Partners
Capitals in 1 : 2 : 2]
Revaluation A/c Dr. 2,000
To Investment A/c 2,000
[Value of investments reduced]
Revaluation A/c Dr. 13,000
To A¡¯s Capital A/c 2,600
To B¡¯s Capital A/c 5,200
To C¡¯s Capital A/c 5,200
(Being profit on revaluation shared in 1 : 2 : 2)
A¡¯s Capital A/c Dr. 1,000
B¡¯s Capital A/c Dr. 2,000
C¡¯s Capital A/c Dr. 2,000
To Goodwill A/c 5,000
[Value of Goodwill reduced]
Profit & Loss Suspense A/c Dr. 500
To C¡¯s Capital A/c 500
[Estimated share of Profit till his date of death
transferred to the decreased partner¡¯s Capital]
C¡¯s Capital A/c Dr. 27,700
To C¡¯s Executors A/c 27,700
[Total dues to the deceased partner
transferred to his Executor¡¯s A/c]
Group-I : Paper-5 : Financial Accounting 23
A, B & C
Capital Accounts
Date Date
2008 Particulars A B C2008 Particulars A B C
Rs. Rs. Rs. Rs. Rs. Rs.
31.3 To Goodwill A/c 1,000 2,000 2,000 31.3. By Balance b/d 10,000 20,000 20,000
To Goodwill A/c 3,333 6,667 " Revaluation A/c 2,600 5,200 5,200
To C's Executors A/c . . 27,700 " Sundry
(Balance transferred) Reserves A/c 2,000 4,000 4,000
To Balance c/d 10,267 20,533 " P & L
14,600 29,200 29,700 Suspense A/c . . 500
14,600 29,200 29,700 14,600 29,200 29,700
A and B
Balance Sheet as at 31.3.2009
Liabilities Amount Amount Assets Amount Amount
Rs. Rs. Rs. Rs.
Capital A/cs: Buildings 35,000
A 10,267 Machinery 25,000
B 20,533 30,800 Moped 3,000
C's Executor's A/c 27,700 Investments 8,000
Bank Loan 30,000 Stock12,000
Sundry Creditors 10,000 Debtors 10,000
Bank 5,000
P & L Suspense A/c 500
98,500 98,500
Q12. A partnership firm was dissolved on 30th June, 2009. Its Balance Sheet on the date of
dissolution was as follows :
Liabilities Rs. Assets Rs.
Capitals : Cash 5,400
Atrik 38,000 Sundry Assets 94,600
Mohit 24,000
Rupa 18,000
Loan A/c . Mohit 5,000
Sundry Creditors 15,000
1,00,000 1,00,000
The assets were realised in instalments and the payments were made on the proportionate capital
basis. Creditors were paid Rs. 14,500 in full settlement of their account. Expenses of realisation
were estimated to be Rs. 2,700 but actual amount spent on this account was Rs. 2,000. This
24 Revisionary Test Paper (Revised Syllabus-2008)
amount was paid on 15th September. Draw up a Memorandum of distribution of Cash, which was
realised as follows :
On 5th July Rs. 12,600
On 30th August Rs. 30,000
On 15th September Rs. 40,000
The partners shared profits and losses in the ratio of 2 : 2 : 1. Give working notes.
Solution :
Statement Showing the Distribution of Cash
(According to Proportionate Capital Method)
Particualrs Creditors Mohit¡¯s Loan Atrik Mohit Rupa
Rs. Rs Rs Rs Rs
A Balance Due 15,000 5,000 33,000 24,000 18,000
B Cash paid (Rs. 5,400 . Rs. 2,700) 2,700 . . . .
C Balance unpaid (A - B) 12,300 5,000 38,000 24,000 18,000
D 1st Instalment of Rs. 12,600 11,800 800 . . .
E Balance unpaid (C - D) 500 4,200 38,000 24,000 18,000
F Less : Written-off 500
G 2nd instalment of Rs. 30,000 4,200 16,320 2,320 7,160
H Balance unpaid (E-F-G) 21,680 21,680 10,840
I 3rd Instalment (Rs. 40,000+Rs. 700) 16,280 16,280 8,140
J Unpaid Balance (H-I) = Loss on
Realisation 5,400 5,400 2,700
Working Notes :
(i) Statement showing the Calculation of Highest Relative Capitals
Particulars Atrik Mohit Rupa
Rs. Rs. Rs.
A Actual Capitals 38,000 24,000 18,000
B Profit-sharing ratio 2 2 1
C Actual Capitals ¡À Profit Sharing Ratio 19,000 12,000 18,000
D Proportionate Capitals taking Shyam¡¯s 24,000 24,000 12,000
E Surplus Capital [A-D] 14,000 Nil 6,000
F Surplus Capital ¡À Profit Sharing Ratio 7,000 . 6,000
G Proportionate capitals taking Mohan¡¯s Capital 12,000 . 6,000
as the basis
H Revised Surplus Capital (E - G) 2,000 . .
Group-I : Paper-5 : Financial Accounting 25
(ii) Distribution of Second Instalment of Rs. 30,000
Creditors Atrik Mohit Rupa
First Rs. 4,200 4,200 . . .
Next Rs. 2,000 (Absolute Surplus) 2,000 . .
Next Rs. 18,000 (Balance of Surplus) 12,000 . 6,000
Balance Rs. 5,800 (2 : 2 : 1) 2,320 2,320 1,160
Total 30,000 4,200 16,320 2,320 7,160
Q13. Rahul, Roshan and Rohan were in partnership sharing profits and losses in the ratio of 3 : 2 : 1
respectively. The partnership was dissolved on 30th June, 2008 when the position was as
follows :
Liabilities Rs. Assets Rs.
Capitals : Cash in hand 28,000
Rahul 1,40,000 Sundry Debtors 2,94,000
Roshan 70,000 Stock in trade 1,12,000
Rohan 14,000
Creditors 2,10,000
4,34,000 4,34,000
There are was bill for Rs. 10,000, due on 30th November, 2008, under discount. It was agreed
that the net realisations should be distributed in their due order (at end of each month) but as
safely as possible. The realisations and expenses were as under :
Date Stock & Debtors Expenses
Rs. Rs.
31st July 84,000 7,000
31st August 1,26,000 5,400
30th September 70,000 4,900
31st October 77,000 3,500
30th November 35,500 3,500
The Stock was completely disposed off and amounts due from debtors were realised, the balance
being irrecoverable. The acceptor of the bill under discount met the billl on the due date. Prepare
a Statement showing the piecemeal distribution of cash according to Maximum Loss Method.
26 Revisionary Test Paper (Revised Syllabus-2008)
Solution :
Statement showing the Distribution of Cash
(According to Maximum Loss Method)
Particulars Creditors Rahul Roshan Rohan
Rs. Rs. Rs. Rs.
A Balance Due 2,10,000 1,40,000 70,000 14,000
B Cash on hand on 30th June paid to creditors 28,000 . . .
C Balance outstanding (A - B) 1,82,000 1,40,000 70,000 14,000
D Cash paid on 31st July 77,000 . . .
E Balance outstanding (C - D) 1,05,000 1,40,000 70,000 14,000
F Rs 1,05,000 paid to creditors on 31st August 1,05,000 . . .
G Balance outstanding (E -F) 1,40,000 70,000 14,000
Balance available for distribution
(Rs. 1,20,600 - Rs. 1,05,000 - Rs. 10,000)
= Rs. 5,600
Less : Maximum loss (Rs. 2,24,000 - Rs. 5,600)
in ratio of 3 : 2 : 1 (1,09,200) (72,800) (36,400)
Balance 30,800 (2,800) (22,400)
Deficiency of Roshan and Rohan¡¯s capital
charged to Rahul (25,200) 2,800 22,400
H Cash paid on 31st August 5,600 . .
I Balance outstanding (G - H) 1,34,400 70,000 14,000
Less : Maximum Loss (Rs. 2,18,400 . Rs. 65,100) (76,650) 51,100) (25,550)
Balance 57,750 18,900 (11,550)
Deficiency of Rohan¡¯s capital charged to
Rahul and Roshan (2 : 1) (7,700) (3,850) 11,550
J Cash paid on 30th September 50,050 15,050 .
K Balance outstanding (I - J) 84350 54,950 14,000
Less : Maximum loss (Rs 1,53,300 . Rs. 73,500) (39,900) (26,600) (13,300)
L Cash paid on 31st October 44,450 28,350 700
M Balance outstanding (K - L) 39,900 26,600 13,300
Less : Maximum loss (Rs 79,800 . Rs. 42,000*) (18,900) (12,600) (6,300)
N Cash paid on 30th November 21,000 14,000 7,000
O Unpaid Balance (M - N) 18,900 12,600 6,300
*Note : Cash available on 30th November = (Rs 35,500 . Rs. 3,500) +Rs. 10,000 (Reserved
for Discounted B/R, now no longer required) = Rs. 42,000.
Group-I : Paper-5 : Financial Accounting 27
Q14. Mr. B and Mr. E are partners sharing Profits and Losses in the ratio of 3:2. On 30th September,
2008 they admit Mr. C as a partner, and the new profit ratio is 2:2:1. C brought in Fixtures
Rs. 3,000 and cash Rs. 10,000, the goodwill being (i) B and E Rs. 20,000 and (ii) C Rs.
10,000 but neither figure is to be brought into the books.
On 31st March, 2009, the partnership is dissolved, B retiring and the other two partners
forming a company called BC Limited with equal capitals, taking over all remaining assets
and liabilities, goodwill being agreed at Rs. 40,000 and brought into books of the company.
B agrees to take over the business car at Rs. 3,700: Plant was sold for Rs. 3,000 being in
excess of requirements. The profit of the two preceding years were Rs. 17,200 and Rs.
19,000 respectively and it was agreed that for the half year ended 30th September, 2008
the net profit was to be taken as equal to the average of the two preceding years and the
current year.
No entries has been made when C entered, except cash. No new book being opened by BC
Company Ltd., B agreed to have Rs. 50,000 as loan to the company, secured by 12%
Debentures. The following is the Trial Balance as on 31st March, 2009.
Debit Credit
Rs. Rs.
Capital Accounts:
B 35,000
E 20,000
C 10,000
Drawing Accounts:
B 6,000
E 5,000
C 2,800
Debtors & Creditors 31,000 12,000
Plant (Book value of plant sold Rs. 4,000) 23,000
Fixtures 7,000
Motor Car 2,700
Stock on 31st March, 09 13,000
Bank16,300
P & L A/c for the year 29,800
1,06,800 1,06,800
Prepare :
(1) Goodwill Adjustment Account
(2) Capital Accounts of Partner
(3) Profit and Loss Appropriation Account
(4) Balance Sheet of BC Ltd. as on 31st March,2009
28 Revisionary Test Paper (Revised Syllabus-2008)
Solution:
(1) Goodwill Adjustment Accoun
Rs. Rs.
2008 2008
30th Sept. 30th Sept.
To Partners¡¯ Capital A/c By Partners¡¯ Capital A/c
(Goodwill raised) (Goodwill written off) (W.N.1)
B 12,000 B 12,000
E 8,000 E 12,000
C 10,000 C 6,000
2009 2009
31st March 31st March
To Partners¡¯ Capital A/cs By Goodwill A/c
(goodwill raised) (Goodwill raised in the
B 16,000 books transferred) 40,000
E 16,000
C 8,000
70,000 70,000
(2) Partners¡¯ Capital Accounts
2009 B E C 2009 B E C
31st March 31st March
To Drawings 6,000 5,000 2,800 By Balance b/d 35,000 20,000 10,000
To Motor Car 3,700 . . By Fixtures
(not recorded
earlier) . . 3,000
To 12% Debentures 50,000 . . By Profit upto
To Goodwill Adjust- 30th Sept ¡®08
ment Account 12,000 12,000 6,000 (W.N.2) 13,200 8,800
To Bank Account 7,620 By Profit for 6
months ended
31st March 2009 3,120 3,120 1,560
To Bank Account . 7,580 . By Goodwill
(WN 3) Adjustment A/c 12,000 8,000 10,000
To Share Capital . 31,340 31,340
By Goodwill
Adjustment A/c 16,000 16,000 8,000
By Bank A/c (W.N.3) . . 7,580
79,320 55,920 40,140 79,320 55,920 40,140
Group-I : Paper-5 : Financial Accounting 29
(3) Profit & Loss Appropriation Account for the year ended 31st March, 2009
Rs. Rs.
To Partners¡¯ Capital Account By Profit & Loss A/c
(Distribution of Profit) (Net profit transferred) 29,800
B 16,320
E 11,920
C 1,560
29,800 29,800
(4) Balance Sheet of BC Ltd.
As on 31st March, 2009
Liabilities Rs. Assets Rs.
Share Capital 62,680 Fixed Assets :
Secured Loan : Goodwill 40,000
12% Debentures 50,000 Plant 19,000
Current Liabilities & Provisions: Fixtures 10,000
Creditors 12,000 Current Assets, Loans &
Advances :
Stock 13,000
Debtors 31,000
Cash at bank (W.N.4) 11,680
1,24,680 1,24,680
Working Notes :
(1) Goodwill Adjustment as on 30th September, 2008
Total B E C
Rs. Rs. Rs. Rs.
Goodwill raised -
B and E (3:2) 20,000 12,000 8,000 .
C 10,000 10,000
30,000
Goodwill written off in the new
profit sharing ratio (2:2:1) 30,000 12,000 12,000 6,000
30 Revisionary Test Paper (Revised Syllabus-2008)
(2) Calculation of half yearly profit : Rs. Rs.
Profit of the preceding two years
(Rs. 17,200 + Rs. 19,000) 36,200
Current year¡¯s profit 29,800
66,000
Profit for six months ended
30th September,2008 (
¡¿ 66,000) 22,000
Profit for next six months ended
31st March, 2009 (Rs. 29,800 . Rs. 22,000) 7,800
(3) Share Capital of BC Ltd:
Total Capital of the firm before conversion -
E 38,920
C 23,760 62,680
E and C should have have equal share in BC Ltd.
C should bring in cash (¡¿ 62,680 . 23,760) 7,580
E should withdraw cash (38,920 .
¡¿ 62,680) 7,580
Bank Account
(4) Rs. Rs.
To Balance b/d 16,300 By B¡¯s Capital Account 7,620
To Plant Account By E¡¯s Capital
(Sale of Plant) 3,000 (Amount withdrawn) 7,580
To Cs capital A/c By Balance c/d 11,680
(Amount brought in) 7,580
26,880 26,880
(5) Profit and loss on sale and takeover of assets : Rs.
Profit on Motor car taken over (Rs. 3,700 . Rs. 2,700) 1,000
Loss on sale of plant (Rs. 4,000 . Rs. 3,000) 1,000
Net effect Nil
Q15. (a) State the Conditions to be fulfilled by a Joint Stock Company to buy-back its equity
shares.
Answer 15. (a)
As per section 77A of the Companies Act, 1956 a joint stock company has to fulfill the following
conditions to buy-back its own equity shares:
(i) The buy-back is authorised by its articles.
(ii) A special resolution* has been passed in general meeting of the company authorising the
buy-back.
Group-I : Paper-5 : Financial Accounting 31
(iii) The buy-back does not exceed 25% of the total paid up capital and free reserves of the
company. Provided the buy.back must not exceed 25% of its total paid up equity capital in
that financial year.
(iv) The ratio of the debt owed by the company is not more than twice the capital and its free
reserves after such buy-back.
(v) All the shares for buy-back are fully paid up.
(vi) The buy-back is made out of the free reserves (which include securities premium) or out of
the proceeds of a fresh issue of any shares or other specified securities.
(vii) The buy-back is completed within 12 months of the passing of the special resolution or a
resolution passed by the Board.
(vii) The buy-back of the shares listed on any recognised stock exchange is in accordance with
the regulations made by the SEBI in this behalf.
(ix) Before making such buy-back, a listed company has to file with the Registrar and the SEBI
a declaration of solvency in the prescribed form.
Q15. (b) Sweat equity shares and conditions, which must be fulfilled by a Joint Stock Company to
issue these shares.
Answer 15. (b)
The Companies (Amendment) Act, 1999 introduced through section 79A a new type of equity
shares called ¡®Sweat Equity Shares. The expression ¡®sweat equity shares¡¯ means equity shares
issued by a company to its employees or directors at a discount or for consideration other than
cash for providing know-how or making available rights in the nature of intellectual property rights
or value additions by whatever name called.
Notwithstanding anything contained in section 79, which deals with the power of a company to
issue shares at a discount, a company may issue sweat equity shares of a class of shares already
issued, if the following conditions are fulfilled, namely :
(i) the issue of sweat equity shares is authorised by a special resolution passed by the company
in the general meeting.
(ii) the resolution specifies the number of shares, current market price, the consideration if
any, and the class or classes of directors or employees to whom such equity shares are to
be issued.
(iii) not less than one year has, at the time of the issue, elapsed since the date on which the
company was entitled to commence business.
(iv) the sweat equity shares of company, whose equity shares are listed on a recognised stock
exchange, are issued in accordance with the regulations made by the Securities and Exchange
Board of India in this behalf. But in the case of company whose equity shares are not listed
on any recognised stock exchange, the sweat equity shares are issued in accordance with
the guidelines as may be prescribed.
All the limitations, restrictions and provisions relating to equity shares are applicable to sweat
equity shares also.
32 Revisionary Test Paper (Revised Syllabus-2008)
Q16. (a) Discuss the payment of dividend on partly paid shares.
Answer 16. (a)
In the case of partly paid-up shares, the dividend is payable either on the nominal, called-up or the
paid-up amount of shares, depending on the provisions in this regard that there may be in the
articles of the company. In the absence of any such provisions, Table A should be applicable. In
such a case the amount of dividend payable will be calculated on the amount paid-up on the
shares, and while doing so, the dates on which the amounts were paid must be taken into account.
Calls paid in advance do not rank for payment of dividend. A company may if so authorised by its
articles, pay a dividend in proportion to the amount paid on each share, where a larger amount is
paid on some shares than on others (Section 93 of the Companies Act, 1956). But where the
articles are silent and Table A has been excluded, the amount of dividend payable will have to be
calculated on the nominal amount of shares. It should, however, be noted that according to Clause
88 of Table A dividends are to be declared and paid according to the amounts paid or credited as
paid on the shares in respect whereof the dividend is paid, but if and so long as nothing is paid upon
any of the shares of the company, dividends may be declared and paid according to the nominal
amount of the shares.
Q16. (b) State the guidelines of SEBI regarding issue of convertible debentures for disclosure and
investor protection.
Answer 16. (b)
SEBI guidelines regarding issue of convertible debentures for disclosure and investor protection
are as follows :
(i) Issue of Fully Convertible Debentures (FCDs) having a conversion period of more than 36
months will not be permissible, unless conversion is made optional with ¡°put¡± and ¡°call¡±
option.
(ii) No company shall make a public issue or rights issue of debts instruments (whether convertible
or not), unless credit rating of not less than investment grade is obtained from not less than
two registered credit rating agencies and disclosed in the offer document. All the credit
ratings including the unaccepted credit ratings shall be disclosed.
(iii) All the credit ratings obtained during the 3 years preceding the public or rights issue of
convertible debentures for any listed security of the issuer company shall be disclosed in the
offer document.
(iv) No company shall issue a prospectus or a letter of offer to the public for subscription of its
debentures without the appointment of a debenture trustees or creation of Debenture
Redemption Reserve, in accordance with the provisions of the Companies Act, 1956. The
names of the debenture trustees shall be stated in the Offer Document and also in all the
subsequent periodical communication. Also a trust deed shall be executed within 3 months
of the closure of the issue.
(v) The merchant banker shall ensure that the security created is adequate to ensure 100%
asset cover for the debentures and is free from any encumbrances and also the necessary
permissions to mortgage the assets or No objection certificate for a second or pari passu
charged in cases where assets are encumbered have been obtained.
(vi) Premium amount on conversion and time of conversion, shall be predetermined by the issuer
company and stated in the prospectus. Interest rates for the above debentures will also be
freely determined by the issuer company.
Group-I : Paper-5 : Financial Accounting 33
(vii) Any conversion in part or whole of the debentures will be optional in the hands of the
debentureholders, if the conversion takes place at or after 18 months from the date of
allotment, but before 36 months.
(viii) Premium amount at the time of conversion for the Partly Convertible Debentures (PCD)
shall be predetermined and stated in the prospectus. Redemption amount, period of maturity,
yield on redemption for the PCDs or NCDs shall also be indicated in the prospectus.
(ix) In case, the non-convertible portions of PCDs or NCDs are to be rolled over with or without
change in the interest rate, a compulsory option should be given to those debentureholders
who want to withdraw and encash from the debenture programme. Roll over shall be done
only in cases where debentureholders have sent their positive consent and not on the basis
of the non-receipt of their negative reply.
(x) Before roll over of any NCDs or non-convertible portion of the PCDs, at least two credit
ratings of not less than investment grade shall be obtained within a period of six months
prior to the due date of redemption and communicated to debentureholders before roll over
and fresh trust deed shall be made.
(xi) Letter of information regarding roll over shall be vetted by SEBI with regard to the credit
ratings, debentureholder¡¯s resolution, option for conversion and such other items which
SEBI may prescribe from time to time.
(xii) The disclosures relating to raising of debentures will contain, amongst other things, the
existing and future equity and long term debt ratio, servicing behaviour on existing debentures,
payment of due interest on due dates on long term loans and debentures, certificate from a
financial institution or bankers about their no objection for a second or pari passu charge
created in favour of the trustees to the proposed debenture issues.
(xiii) SEBI may prescribe additional disclosure requirement from time to time, after due notice.
Q17. The Balance sheet of XYZ Ltd. as at 31st December, 2008 inter alia includes the following :
Rs.
50,000 8% Preference shares of Rs. 100 each Rs. 70 paid up 35,00,000
1,00,00 Equity shares of Rs. 100 each fully paid up 1,00,00,000
Securities premium 5,00,000
Capital redemption reserve 20,00,000
General reserve 50,00,000
Under the terms of their issue, the preference shares are redeemable on March 31, 2009 at a
premium of 5%. In order to finance the redemption, the company makes a right issue of 50,000
equity shares of Rs. 100 each at Rs. 20 being payable on application, Rs. 35 (including premium)
on allotment and the balance on January 1, 2010. The issue was fully subscribed and allotment
made on March 1, 2009. The monies due on allotment were received by March 30, 1999.
The preference shares were redeemed after fulfilling the necessary conditions of Section 80 of
the Companies Act, 1956. The company decided to make the minimum utilisation of general
reserve.
You are asked to pass the necessary jounal entries and show the relevant extracts from the
Balance Sheet as on March 31, 2009 with the corresponding figures as on 31st December, 2008.
34 Revisionary Test Paper (Revised Syllabus-2008)
Solution:
XYZ Ltd.
Journal Entries
Dr. Cr.
Rs. ¡®000 Rs. ¡®000
8% Preference Share Final Call Account Dr. 15,00
To 8% Preference Share Capital Account 15,00
(Being the final call made on 50,000 preference shares
@ Rs. 30 each to make them fully paid up)
Bank Account Dr. 15,00
To 8% Preference Share Final Call Account 15,00
(Being the final call amount received on 50,000
preference shares @ Rs. 30 each)
Bank Account Dr. 10,00
To Equity Share Application Account 10,00
(Being the application money received on 50,000
equity shares @ Rs. 20 per share)
Equity Share Application Account Dr. 10,00
To Equity Share Capital Account 10,00
(Being the application money on 50,000 equity shares
transferred to equity share capital account vide Board¡¯s
resolution dated...)
Equity Share Allotment Account Dr. 17,50
To Equity Share Capital Account 12,50
To Securities Premium Account 5,00
(Being the amount due on 50,000 equity shares @ Rs. 35
per share [including premium Rs.] 10 vide Board¡¯s
resolution dated...)
Bank Account Dr. 17,50
To Equity Share Allotment Account 17,50
(Being the allotment money received on 50,000 equity
shares @ Rs. 35 per share)
8% Preference Share Capital Account Dr. 50,00
Premium on Redemption of Preference Shares Account Dr. 2,50
To Preference Shareholders Account 52,50
(Being the amount payable to preference share holders
on redemption)
Group-I : Paper-5 : Financial Accounting 35
Dr. Cr.
Rs. ¡®000 Rs. ¡®000
Preference Shareholders Account Dr. 52,50
To Bank Account 52,50
(Being the payment made to preference shareholders)
Securities Premium Account Dr. 2,50
To Premium on Redemption of Preference Shares Account 2,50
(Being the premium payable on redemption of preference
shares charged to share premium account)
General Reserve Dr. 27,50
To Capital Redemption Reserve 27,50
(Being the amount transferred to capital redemption
reserve on redemption of preference shares for the
balance not covered by proceeds of fresh issue of shares)
Balance Sheet of XYZ Limited
As at 31st March, 1999 (after redemption of preference shares)
(Relevant extracts)
Amount Amount
Rs. (¡®000) Rs. (¡®000)
As on As on
31.3.99 31.12.98
1. Sources of funds
Shareholders¡¯ funds :
(a) Share Capital
Issued, subscribed and paid-up
1,00,000 equity shares of Rs. 100 each, fully paid up 1,00,00 1,00,00
50,000 equity shares of Rs. 100 each,
Rs. 45 called up and paid up 22,50 .
50,000, 8% Redeemable preference shares of
Rs. 100 each, Rs. 70 called-up and paid-up
(redeemed on 31st March, 1999) . 35,00
1,22,50 1,35,00
(b) Reserves and Surplus :
Capital redemption reserve 47,50 20,00
Securities premium account 7,50 5,00
General reserve 22,50 50,00
77,50 75,00
36 Revisionary Test Paper (Revised Syllabus-2008)
The cash and bank balance will be decreased by Rs. 10,00,000 on 31.3.09 as compared to the
balance on 31.12.08.
Working Notes : Rs. ¡®000
(i) Transfer to Capital Redemption Reserve
Nominal value of preference shares redeemed (Rs. 100 ¡¿ 50,000) 50,00
Less : Proceeds of fresh equity issue [(Rs. 20 + 25) ¡¿ 50,000)] 22,50
Transfer to capital redemption reserve 27,50
(ii) Capital Redemption Reserve as on 31.3.09
Balance as on 31.12.08 20,00
Add : Transfer from general reserve 27,50
Balance as on 31.3.09 47,50
(iii) General reserve as on 31.3.09
Balance as on 31.12.08 50,00
Less : Transfer to capital redemption reserve 27,50
Balance as on 31.3.09 22,50
(iv) Securities premium as on 31.3.09
Balance as on 31.12.08 5.00
Add : Amount received @ Rs. 10 per share on fresh issue of 50,000
equity shares 5.00
Less : Premium on redemption of preference shares 2.50
Balance as on 31.3.09 7.50
(v) Change in cash and bank balance
Receipts : (31.12.08 - 31.3.09)
Application money on 50,000 equity shares @ Rs. 20 per share 10.00
Allotment money on 50,000 equity shares @ Rs. 35 per share 17.50
Final call on 50,000, 8% Preference shares @ Rs. 30 per share 15.00
42.50
Payments :
Amount paid to preference shareholders on redemption 52.50
Reduction in cash and bank balance 10.00
Q18. (a) From the following particulars of Ganga Limited, you are required to calculate the
managerial remuneration in the following situation.
(i) There is only one whole time director.
(ii) There ar two whole time directors.
(iii) There are two whole time directors, a part time director and a Manager.
Group-I : Paper-5 : Financial Accounting 37
Rs.
Net profit before provision for income-tax and managerial
remuneration, but after depreciation and provision for repairs 8,70,410
Depreciation provided in the books 3,10,000
Provision for repairs of machinery during the year 25,000
Depreciation allowable under Schedule XIV 2,60,000
Actual expenditure incurred on repairs during the year 15,000
Solution:
Sections 198 and 309 of the Comapnies Act, 1956 prescribe the maximum percentage of profit
that can be paid as managerial remuneration. For this purpose, profit is to be calculated in the
manner as specified in Section 349.
Calculation of net profit u/s 349 of the Companies Act, 1956
Rs. Rs.
Net profit before provision for income-tax and managerial
remuneration, but after depreciation and provision for repairs 8,70,410
Add back : Depreciation provided in the books 3,10,000
Provision for repairs of machinery 25,000 3,35,000
12,05,410
Less : Depreciation allowable under Schedule XIV 2,60,000
Actual expenditure incurred on repairs 15,000 2,75,000
Profit under section 349 9,30,410
Calculation of managerial remuneration
(i) There is only one whole time director
Managerial remuneration = 5% of Rs. 9,30,410
= Rs. 46,520.50
(ii) There are two whole time directors
Managerial remuneration = 10% of Rs. 9,30,410
= Rs. 93,041
(iii) There are two whole time directors, a part time director and a manager
Managerial remuneration = 11% of Rs. 9,30,410
= Rs. 1,02,345.10
Q18. (b) The trial balance of Complex Ltd. as at 31st March, 2009 shows the following items :
Dr. Cr.
Rs. Rs.
Advance payment of income-tax 2,20,000 .
Provision for income-tax for the year ended 31.3.08 . 1,20,000
38 Revisionary Test Paper (Revised Syllabus-2008)
The following further informations are given :
(i) Advance payment of income-tax includes Rs. 1,40,000 for 2007-08.
(ii) Actual tax liability for 2007-08 amounts to Rs. 1,52,000 and no effect for the same has so
far been given in accounts.
(iii) Provision for income-tax has to be made for 2008-09 for Rs. 1,60,000.
You are required to prepare (a) provision for income-tax account, (b) advance payment of incometax
account, (c) liabilities for taxation account and also show, how the relevant items will appear
in the profit and loss account and balance sheet of the Company.
Solution:
Complex Ltd.
(a) Provision for Income Tax (2007-08) Account
Dr. Cr.
Rs. Rs.
31.3.09 To Advance Payment of 1.4.08 By Balance b/d 1,20,000
Income-tax A/c 1,40,000 31.3.09 By Profit and Loss A/c 32,000
To Liability for
Taxation A/c 12,000
1,52,000 1,52,000
Provision for Income-tax (2008-09) Account
Rs. Rs.
31.3.09 To Balance c/d 1,60,000 31.3.09 By Profit and Loss A/c 1,60,000
1,60,000 1,60,000
(b) Advance Payment of Income Tax Account
Rs. Rs.
31.3.09 To Balance b/d 2,20,000 31.3.09 By Provision for Incometax
(2007-08) A/c 1,40,000
By Balance c/d 80,000
2,20,000 2,20,000
(c) Liability for Taxation Account
Rs. Rs.
31.3.09 To Balance c/d 12,000 31.3.09 By Provision for Incometax
A/c 12,000
12,000 12,000
Group-I : Paper-5 : Financial Accounting 39
Profit and Loss Account
for the year ended 31st March, 2009 (Extracts)
Rs. Rs.
Profit before Taxation ....
Less : Taxation for the year 1,60,000
Taxation adjustment of previous year 32,000 1,92,000
Net Profit ....
Balance Sheet of Complex Ltd.
As at 31st March, 2009 (Extracts)
Liabilities Rs. Assets Rs.
Current Liabilities and Provisions Current Assets, Loans and Advances
A. Current Liabilities B. Loans and Advances
Liability for Taxation (1996-97) 12,000 Advance payment of
B. Provisions Income-tax 80,000
Provision for Income-tax 1,60,000
Q19. Fruit Juice Ltd., Mumbai has factories at Ratnagiri (alphonso mango pulp) and Nagpur (Orange
juice). During the year ended 31st March, 2009, the following locationwise revenue
statements were furnished by the two factories (from which the total column has been
compiled) :
Ratnagiri Nagpur Total
Rs. Rs. Rs.
Opening stock :
Work in process 24,000 12,000 36,000
Finished goods 8,000 2,000 10,000
32,000 14,000 46,000
Raw material consumption 25,00,000 10,00,000 35,00,000
Employee cost 5,00,000 6,00,000 11,00,000
Power and Fuel 1,00,000 50,000 1,50,000
Consumable stores 15,000 7,000 22,000
Rates and taxes 14,000 9,000 23,000
Repairs to factory :
Building 4,000 5,000 9,000
Machinery 80,000 50,000 1,30,000
Other assets 3,000 1,000 4,000
Other expenses 65,000 55,000 1,20,000
Depreciation 1,00,000 90,000 1,90,000
34,13,000 18,81,000 52,94,000
Less : Closing stock
Work in process 28,000 13,000 41,000
Finished goods 5,000 8,000 13,000
33,000 21,000 54,000
Cost of goods transferred to marketing division 33,80,000 18,60,000 52,40,000
40 Revisionary Test Paper (Revised Syllabus-2008)
The marketing division furnishes you with the following information of its productwise revenue
statement for the year ended 31st March, 2009 (from which the total column has been compiled) :
Mango pulp Orange juice Total
Opening stock : 12,000 5,000 17,000
Receipt during the year out of :
Last year¡¯s despatch from factory 10,000 5,000 15,000
Current year¡¯s despatch from factory 33,65,000 18,50,000 52,30,000
33,75,000 18,55,000 52,30,000
Transport ¡°in¡± cost from factory 50,000 60,000 1,10,000
34,37,000 19,20,000 53,57,000
Less : Closing stock 7,000 10,000 17,000
34,30,000 19,10,000 53,40,000
Sales commission 5,00,000 2,50,000 7,50,000
Sales tax 4,00,000 1,25,000 5,25,000
Profit 6,70,000 2,15,000 8,85,000
Sales 50,00,000 25,00,000 75,00,000
You are asked to prepare sectional and consolidated revenue statement for the year ended 31st
March, 2009 for consideration of the board of directors and presentation to the members of Fruit
Juice Ltd. Also work out the percentage of net profit to sales.Show your working, if any.
Solution:
Revenue Statement of Fruit Juice Ltd. (Sectional and Consolidated)
for the year ended 31st March, 2009
Mango Pulp Orange Juice Total
Rs. ¡®000 Rs. ¡®000 Rs. ¡®000
Sales 5,000 2,500 7,500
Add : Excess of closing inventory over
opening inventory (Working note 1) 1 17 18
Gross revenue 5,001 2,517 7,518
Less : Manufacturing and other expenses 4,231 2,212 6,443
(Working note 2)
Profit before depreciation 770 305 1,075
Less : Depreciation 100 90 190
Net Profit 670 215 885
Mango Pulp Orange Juice Total
Percentage of net profit to sales 13.4% 8.6% 11.8%
Group-I : Paper-5 : Financial Accounting 41
Working Notes :
(1) Excess of closing inventory over opening inventory
(a) Mango Pulp Orange Juice Total
Rs. ¡®000 Rs. ¡®000 Rs. ¡®000
Opening Stock
Finished goods :
At factory 8 2 10
In transit (received during the year by
marketing division) 10 5 15
With marketing division 12 5 17
30 12 42
Work in process 24 12 36
Total 54 24 78
(b) Mango Pulp Orange Juice Total
Rs. ¡®000 Rs. ¡®000 Rs. ¡®000
Closing Stock
Finished goods :
At factory 5 8 13
In transit* 15 10 25
With marketing division 7 10 17
27 28 55
Work in process 28 13 41
55 41 96
(c) Closing Stock5 5 41 96
Less :Opening stock5 4 24 78
Excess of closing stock over opening stock 1 17 18
*Goods sent by factory 3,380 1,860 5,240
Less : Received by marketing division 3,365 1,850 5,215
Finished goods in transit 15 10 25
(2) Manufacturing and other costs
Mango Pulp Orange Juice Total
Rs. ¡®000 Rs. ¡®000 Rs. ¡®000
Manufacturing costs:
Raw Material consumption 2,500 1,000 3,500
Employee cost 500 600 1,100
Power and fuel 100 50 150
Consumable stores 15 7 22
Rates and taxes 14 9 23
42 Revisionary Test Paper (Revised Syllabus-2008)
Repairs : Building 4 5 9
Machinery 80 50 130
Other assets 3 1 4
Other costs :
Transport 50 60 110
Sales commission 500 250 750
Sales tax 400 125 525
Other expenses 65 55 120
4,231 2,212 6,443
Q20. (i) A major fire has damaged the assets in a factory of a limited company on 2nd April-two
days after the year end closure of account. The loss is estimated at Rs. 20 crores out of
which Rs. 12 crores will be recoverable from the insurers. Explain briefly how the loss
should be treated in the final accounts for the previous year.
(ii) There is a sales tax demand of Rs. 2.50 crores against a company relating to prior years
against which the company has gone on appeal to the appellate authority in the department.
The grounds of appeal deal with points covering Rs. 2 crores of the demand. State how
the matter will have to be dealt with in the final accounts for the year.
Answer :
(i) The loss due to break out of fire is an example of event occurring after the balance sheet
date that does not relate to conditions existing at the balance sheet date. It has not affected
the financial position as on the date of the balance sheet and therefore requires no specific
adjustments in the financial statements. However, paragraph 8.6 of AS-4 states that disclosure
is generally made of events in subsequent periods that represent unusual changes affecting
the existence or substratum of the enterprise at the balance sheet date. In the given case,
the loss of assets in a factory is considered to be an event affecting the substratum of the
enterprise after the balance sheet date. Hence, as recommended in paragraph 15 of AS-4,
disclosure of the event should be made in the report of the approving authority that represent
material changes and commitments affecting the financial position of the enterprise.
(ii) The undisputed part of sales tax liability of Rs. 0.50 crore should be considered as actual
liability and adequately provided for. The Institute of Chartered Accountants of India has
issued Accounting standard 29 on ¡°Provisions Contingent Liabilities and Contingent Assets¡±
(comes into effect in respect of accounting periods commencing on or after 1.4.2004).
According to the standard, an enterprise should not recognise a contingent liability but
should disclose it, as required by paragraph 68, unless the possibility of an outflow of resources
embodying economic benefits is remote. Accordingly the company should disclose the disputed
part of sales tax liability of Rs. 2 crore as contingent liability in their financial statements of
the year. However, the above disclosed contingent liability should be reviewed continuosly
and if it becomes probable that an outflow of future economic benefit will be required , then
recognise the contingent liability as a provision.
Q21. Jagannath Ltd. had made a rights issue of shares in 2006. In the offer document to its
members, it had projected a surplus of Rs. 40 crores during the accounting year to end on
31st March, 2009. The draft results for the year, prepared on the hitherto followed accounting
Group-I : Paper-5 : Financial Accounting 43
policies and presented for perusal of the board of directors showed a deficit of Rs. 10
crores. The board in consultation with the managing director, decided on the following :
(i) Value year-end inventory at works cost (Rs. 50 crores) instead of the hitherto method of
valuation of inventory at prime cost (Rs. 30 crores).
(ii) Provide depreciation for the year on straight line basis on account of substantial additions
in gross block during the year, instead of on the reducing balance method, which was
hitherto adopted. As a consequence, the charge for depreciation at Rs. 27 crores is
lower than the amount of Rs. 45 crores which would have been provided had the old
method been followed, by Rs. 18 cores.
(iii) Not to provide for ¡°after sales expenses¡± during the warranty period. Till the last year,
provision at 2% of sales used to be made under the concept of ¡°matching of costs
against revenue¡± and actual expenses used to be charged against the provision. The
board now decided to account for expenses as and when actually incurred. Sales during
the year total to Rs. 600 crores.
(iv) Provide for permanent fall in the value of investments - which fall had taken place over
the past five years - the provision being Rs. 10 crores.
As chief accountant of the company, you are asked by the managing director to draft the notes on
accounts for inclusion in the annual report for 2008-09
Answer :
As per AS 1 ¡°Any change in the accounting policies which has a material effect in the current
period or which is reasonably expected to have a material effect in later periods should be disclosed.
In the case of a change in accounting policies which has a material effect in the current period, the
amount by which any item in the financial statements is affected by such change should also be
disclosed to the extent ascertainable. Where such amount is not ascertainable, wholly or in part,
the fact should be indicated. Accordingly, the notes on accounts should properly disclose the
change and its effect.
Notes on Accounts :
(i) During the year inventory has been valued at factory cost, against the practice of valuing it
at prime cost as was the practice till last year. This has been done to take cognisance of the
more capital intensive method of production on account of heavy capital expenditure during
the year. As a result of this change, the year-end inventory has been valued at Rs. 50 crores
and the profit for the year is increased by Rs. 20 crores.
(ii) In view of the heavy capital intensive method of production introduced during the year, the
company has decided to change the method of providing depreciation from reducing balance
method to straight line method. As a result of this change, depreciation has been provided
at Rs. 27 crores which is lower than the charge which would have been made had the old
method and the old rates been applied, by Rs. 18 crores. To that extent, the profit for the
year is increased.
(iii) So far, the company has been providing 2% of sales for meeting ¡°after sales expenses
during the warranty period. With the improved method of production, the probability of
defects occurring in the products has reduced considerably. Hence, the company has decided
not to make provision for such expenses but to account for the same as and when expenses
are incurred. Due to this change, the profit for the year is increased by Rs. 12 crores than
would have been the case if the old policy were to continue.
44 Revisionary Test Paper (Revised Syllabus-2008)
(iv) The company has decided to provide Rs. 10 crores for the permanent fall in the value of
investments which has taken place over the period of past five years. the provision so made
has reduced the profit disclosed in the accounts by Rs. 10 crores.
Q22. (a) When Capitalisation of borrowing cost should cease as per Accounting Standard 16?
(b) Define a ¡°Business Segment¡± and a ¡°Geographical Segment¡± as per Accounting
Standard 17.
(c) Briefly describe, how do you calculate ¡°Diluted Earnings per Share¡± as per Accounting
Standard 20.
(d) Briefly describe the disclosure requirements for ¡°Deferred Tax Assets¡± and ¡°Deferred
Tax Liabilities¡± as per Accounting Standard 22.
(e) Write short note on Sale and Lease Back Transactions as per Accounting Standard 19.
Answer :
(a) Capitalisation of borrowing costs should cease when substantially all the activities necessary
to prepare the qualifying asset for its intended use or sale are complete.
An asset is normally ready for its intended use or sale when its physical construction or
production is complete even though routine administrative work might still continue. If
minor modifications such as the decoration of a property to the user¡¯s specification, are all
that are outstanding, this indicates that substantially all the activities are complete.
When the construction of a qualifying asset is completed in parts and a completed part is
capable of being used while construction continues for the other parts, capitalisation of
borrowing costs in relation to a part should cease when substantially all the activities
necessary to prepare that part for its intended use or sale are complete.
(b) A Business Segment: A business segment is a distinguishable component of an enterprise
that is engaged in providing an individual product or service or a group of related products or
services and that is subject to risks and returns that are different from those of other
business segments. Factors that should be considered in determining whether products or
services are related include:
(a) the nature of the products or services;
(b) the nature of the production processes;
(c) the type or class of customers for the products or services;
(d) the methods used to distribute the products or provide the services and
(e) if applicable, the nature of the regulatory environment, for example, banking, insurance
or public utilities.
A geographical segment: A geographical segment is a distinguishable component of an
enterprise that is engaged in providing product or services within a particular economic
environment and that is subject to risks and returns that are different from those of components
operating in other economic environments. Factors that should be considered in identifying
geographical segments include :
(a) similarity of economic and political conditions;
(b) relationships between operations in different geographical areas;
(c) proximity of operations;
Group-I : Paper-5 : Financial Accounting 45
(d) special risks associated with operations in a particular area;
(e) exchange control regulations; and
(f) the underlying currency risks.
(c) For the purpose of calculating diluted earnings per share, the net profit or loss for the period
attributable to equity shareholders and the weighted average number of shares outstanding
during the period should be adjusted for the effects of all dilutive potential equity shares.
The amount of net profit or loss for the period attributable to equity shareholders should be
adjusted, after taking into account any attributable change in tax expense for the period.
The number of equity shares should be the aggregate of the weighted average number of
equity shares (as per paragraphs 15 and 22 of AS 20) and the weighted average number of
equity shares which would be issued on the conversion of all the dilutive potential equity
shares into equity shares. Dilutive potential equity shares should be deemed to have been
converted into equity shares at the beginning of the period or, if issued later, the date of the
issue of the potential equity shares.
An enterprise should assume the exercise of dilutive options and other dilutive potential
equity shares of the enterprise. The assumed proceeds from these issues should be considered
to have been received from the issue of shares at fair value. The difference between the
number of shares issuable and the number of shares that would have been issued at fair
value should be treated as an issue of equity shares for no consideration.
(d) (i) An enterprise should offset deferred tax assets and deferred tax liabilities if:
(a) the enterprise has a legally enforceable right to set off assets against liabilities
representing current tax, and
(b) the deferred tax assets and the deferred tax liabilities relate to taxes on income
levied by the same governing taxation laws.
(ii) Deferred tax assets and liabilities should be distinguished from assets and liabilities
representing current tax for the period. Deferred tax assets and liabilities should be
disclosed under a separate heading in the balance sheet of the enterprise, separately
from current assets and current liabilities.
(iii) The break-up of deferred tax assets and deferred tax liabilities into major components of
the respective balances should be disclosed in the notes to accounts.
Q23. (a) X Co. Limited purchased goods at the cost of Rs.40 lakhs in October, 2008. Till March,
2009, 75% of the stocks were sold. The company wants to disclose closing stock at
Rs.10 lakhs. The expected sale value is Rs.11 lakhs and a commission at 10% on sale is
payable to the agent. Advise, what is the correct closing stock to be disclosed as at
31.3.2009.
(b) Explain the ¡®Accounting of Revaluation of Assets¡¯ with reference to AS 10.
(c) Arjun Ltd. sold farm equipments through its dealers. One of the conditions at the time of
sale is, payment of consideration in 14 days and in the event of delay interest is chargeable
@ 15% per annum. The Company has not realized interest from the dealers in the past.
However, for the year ended 31.3.2009, it wants to recognise interest due on the balances
due from dealers. The amount is ascertained at Rs.9 lakhs. Decide whether the income
by way of interest from dealers is eligible for recognition as per AS 9.
46 Revisionary Test Paper (Revised Syllabus-2008)
(d) AB Ltd. launched a project for producing product X in October, 2007. The Company
incurred Rs. 20 lakhs towards Research and Development expenses upto 31st March,
2009. Due to prevailing market conditions, the Management came to conclusion that
the product cannot be manufactured and sold in the market for the next 10 years. The
Management hence wants to defer the expenditure write off to future years.
Advise the Company as per the applicable Accounting Standard.
Answer :
(a) As per Para 5 of AS 2 ¡°Valuation of Inventories¡±, the inventories are to be valued at lower
of cost and net realizable value.
In this case, the cost of inventory is Rs.10 lakhs. The net realizable value is 11,00,000 ¢¥
90% = Rs.9,90,000. So, the stock should be valued at Rs.9,90,000.
(b) As per Para 30 of AS 10 ¡°Accounting for Fixed Assets¡±, an increase in net book value
arising on revaluation of fixed assets should be credited to owner¡¯s interests under the head
of ¡®revaluation reserve, except that, to the extent that such increase is related to and not
greater than a decrease arising on revaluation previously recorded as a charge to the profit
and loss statement, it may be credited to the profit and loss statement. A decrease in net
book value arising on revaluation of fixed assets is charged directly to profit and loss statement
except that to the extent such a decrease is related to an increase which was previously
recorded as a credit to revaluation reserve and which has not been subsequently reversed
or utilized , it may be charged directly to that account.
(c) As per AS 9 ¡°Revenue Recognition¡±, where the ability to assess the ultimate collection with
reasonable certainty is lacking at the time of raising any claim, the revenue recognition is
postponed to the extent of uncertainty inverted. In such cases, the revenue is recognized
only when it is reasonably certain that the ultimate collection will be made.
In this case, the company never realized interest for the delayed payments make by the
dealers. Hence, it has to recognize the interest only if the ultimate collection is certain. The
interest income hence is not to be recognized.
(d) As per Para 41 of AS 26 ¡°Intangible Assets¡±, expenditure on research should be recognized
as an expense when it is incurred. An intangible asset arising from development (or from
the development phase of an internal project) should be recognized if, and only if, an enterprise
can demonstrate all of the conditions specified in para 44 of the standard. An intangible
asset (arising from development) should be derecognised when no future economic benefits
are expected from its use according to para 87 of the standard. Therefore, the manager
cannot defer the expenditure write off to future years.
Hence, the expenses amounting Rs. 20 lakhs incurred on the research and development
project has to be written off in the current year ending 31st March, 2009.
Q24. A head office sends goods to its branch at 20% less than the list price. Goods are sold to
customers at cost plus 100%. From the following particulars, ascertain the profit made at
the head office :
Head Office Branch
Rs. Rs.
Purchases2,00,000 .
Goods sent to/Received by Branch (Invoice Price) 80,000 .
Sales to Customers 1,70,000 80,000
Group-I : Paper-5 : Financial Accounting 47
Solution :
Dr. Branch Trading Profit and Loss Account Cr.
Particulars Rs. Particulars Rs.
To Goods Sent to Branch A/c 80,000 By Sales 80,000
To Retail Profit t/f to Branch By Closing Stock
P & L A/c (b.f.) 16,000 [See working note (ii)] 16,000
96,000 96,000
To Net Profit subject to By Gross Profit b/d 16,000
other Branch Expenses 16,000
16,000 16,000
Dr. Branch Stock Reserve Account Cr.
Particulars Rs. Particulars Rs.
To Balance c/d 6,000 By H.O. P & L A/c 6,000
(Provision for Unrealised Profit
on Closing Stock)
(Rs. 16,000¡¿60/160)
6,000 6,000
Dr. H. O. Trading and Profit & Loss Account Cr.
Particulars Rs. Particulars Rs.
To Purchases 2,00,000 By Sales @ Rs. 200 1,70,000
To Gross Profit t/f to P & L A/c 1,15,000 By Goods sent to whole sale
Branch @ Rs. 160 80,000
By Closing Stock
[See Working Note (iii)] 65,000
3,15,000 3,15,000
To Stock Reserve on
Branch Stock (16,000¡¿60/160) 6,000 By Gross Profit b/d 1,15,000
To Net Profit 1,25,000 By Branch P & L A/c 16,000
1,31,000 1,31,000
Working Notes :
(i) Calculation of Cost-price relationship Rs.
Cost Price 100
Add : Wholesale Profit (balancing figure) 60
Wholesale Price 160
Add : Retail Profit (20% of Rs. 200) 40
List Price 200
48 Revisionary Test Paper (Revised Syllabus-2008)
(ii) Closing Stock at Branch = Opening Stock (at W.P.) + Goods sent (at W.P.) . Wholesale
price of goods sold
= Nil + Rs. 80,000 . (80,000 ¡¿ 160/160) =16,000
(iii) Closing Stock at H.O. = Opening Stock (at cost) + Purchases (at cost) . Cost of Goods
sent to branch . Cost of Goods sold at H.O.
Nil + Rs. 2,00,000 . (Rs. 80,000 ¡¿ 100/160) .
(Rs. 1,70,000¡¿ 100/200)=Rs. 65,000
Q25. Journalise the following transactions in the books of Head Office. Delhi Branch and Agra
Branch :
(a) Goods worth Rs. 50,000 are supplied by Delhi Branch to Agra Branch under the instructions
of Head Office. (b) Delhi Branch draws a bill receivable for Rs 40,000 on Agra Branch which
sends its acceptance. (c) Delhi Branch received Rs 10,000 from Agra Branch. (d) Goods
worth Rs. 20,000 were returned by a customer of Agra Branch to Delhi Branch. (e) Agra
Branch collected Rs 20,000 from a customer of Delhi Branch.
Solution :
Journal of Head Office
Particulars L. F. Dr. (Rs) Cr. (Rs)
(a) Agra Branch A/c Dr. 50,000
To Delhi Branch A/c 50,000
(Being the goods supplied by Delhi Branch to Agra Branch)
(b) Delhi Branch A/c Dr. 40,000
To Agra Branch A/c 40,000
(Being a B/R drawn by Delhi upon Agra Branch)
(c) Delhi Branch A/c Dr. 10,000
To Agra Branch A/c 10,000
(Being Cash sent by Agra Branch to Delhi Branch)
(d) Delhi Branch A/c Dr. 20,000
To Agra Branch A/c 20,000
(Being the goods returned by customer of Agra
Branch to Delhi Branch)
(e) Agra Branch A/c Dr. 20,000
To Delhi Branch A/c 20,000
(Being the Cash collected by Agra Branch from
a customer of Delhi Branch
Group-I : Paper-5 : Financial Accounting 49
Journal of Agra Branch
Particulars L. F. Dr. (Rs) Cr. (Rs)
(a) Goods Received from Branch A/c Dr. 50,000
To Head Office A/c 50,000
(Being goods received from Delhi Branch)
(b) Head Office A/c Dr. 40,000
To B/P A/c 40,000
(Being a B/P accepted for Delhi Branch)
(c) Head Office A/c Dr. 10,000
To Cash A/c 10,000
(Being cash sent to Delhi Branch)
(d) Head Office A/c Dr. 20,000
To Debtors A/c 20,000
(Being goods returned by customer to Delhi Branch)
(e) Cash A/c Dr. 20,000
To Head Office A/c 20,000
(Being cash received from a customer of Delhi Branch)
Journal of Delhi Branch
Particulars L. F. Dr. (Rs) Cr. (Rs)
(a) H.O. A/c Dr. 50,000
To Goods sent to Branch A/c 50,000
(Being the goods supplied to Agra Branch)
(b) Bills Receivable A/c Dr. 40,000
To H.O. A/c 40,000
(Being the acceptance of a B/R received from Agra Branch)
(c) Cash A/c Dr. 10,000
To H.O. A/c 10,000
(Being the cash received from Agra Branch)
(d) Goods Sent to Branch A/c Dr. 20,000
To H.O. A/c 20,000
(Being the goods received from a
customer of Agra Branch)
(e) H.O. A/c Dr. 20,000
To Debtors A/c 20,000
(Being the cash collected by Agra Branch
from our customer)
50 Revisionary Test Paper (Revised Syllabus-2008)
Q26. Z Ltd., has 3 departments, X, Y, Z. The following information is provided :
X Y Z
Rs. Rs. Rs.
Opening Stock 3,000 4,000 6,000
Consumption of direct materials 8,000 12,000 .
Wages 5,000 10,000 .
Closing Stock4,000 14,000 8,000
Sales . . 34,000
Stock of each department is valued at cost to the department connected. Stocks of X department
are transferred to Y at a margin of 50% above departmental cost. Stocks of Y department are
transferred to Z department at a margin of 10% above departmental cost. Other expenses were :
Salaries Rs. 2,000, Printing & Stationery Rs. 1,000, Rent Rs. 6,000, Interest paid Rs. 4,000,
Depreciation Rs. 3,000, Allocate expenses in the ratio of departmental gross profit. Opening
figures of reserve for unrealised profits on departmental stocks were : Department Y Rs. 1,000 ;
Department Z Rs. 2,000.
Required : Prepare Departmental Trading and Profit & Loss Accounts for the year ended March
31, 2009.
Solution :
Departmental Trading and Profit & Loss Acconts
Dr. for the year ended 31st March, 2009 Cr.
Particulars X Y Z Total Particulars X Y Z Total
Rs. Rs. Rs. Rs. Rs. Rs. Rs. Rs
To Opening Stock : 3,000 4,000 6,000 13,000 By Internal b/d 18,000 33,000 . 51,000
To Direct Material By Sales . . 34,000 34,000
considered 8,000 12,000 . 20,000 By Closing Stock 4,000 14,000 8,000 26,000
To Wages 5,000 10,000 . 15,000
To Internal Transfer . 18,000 33,000 51,000
To Gross Profit c/d 6,000 3,000 3,000 12,000
22,000 47,000 42,000 1,11,000 22,000 47,000 42,000 1,11,000
To Salaries 1,000 500 500 2,000 By Gross Profit
To Printing & b/d 6,000 3,000 3,000 12,000
Stationery 500 250 250 1,000 By Net Loss c/d 2,000 1,000 1,000 4,000
To Rent 3,000 1,500 1,500 6,000
To Depreciation 1,500 750 750 3,000
To Interest paid 2,000 1,000 1,000 4,000
8,000 4,000 4,000 16,000 8,000 4,000 4,000 16,000
To Net Loss b/d 4,000 By Provision for
To Provision for unrealised
unrealised profit profit on
on Closing Stock 3,918 Opening Stock 3,000
By Balance transferred
to
P & L A/c 4,918
7,918 7,918
Group-I : Paper-5 : Financial Accounting 51
Working Notes :
(i) FIFO method for stock issue has been assumed. Alternatively this question could have
been solved by assuming other methods for stock issue like LIFO Basis, Weighted Average
basis, etc.
(ii) Calculation of unrealised profit on Closing Stock of Deptt. Y Rs.
Current cost incurred by Dept. Y (Rs. 12,000 + Rs. 10,000 + Rs. 18,000) 40,000
Profit included in Above (Rs. 18,000¡¿50/150) 6,000
Profit included in Closing Stock of Rs. 14,000
(Rs. 6,000¡¿Rs. 14,000/Rs. 40,000) 2,100
(iii) Calculation of unrealised profit on Closing Stock of Dept C Rs.
Current Cost incurred by Dept. Z 33,000
Profit of Dept. B included in above (Rs. 33,000¡¿ 10/110) 3,000
Cost element of Dept. Y included in current cost (Rs. 33,000 . 3,000) 30,000
Profit of Dept. a included in above cost (Rs. 6,000 ¡¿ Rs. 30,000/Rs. 40,000)
Total Profit included in current cost of Dept. Z (Rs. 3,000 + Rs. 4,500) 7,500
Unrealised profit included in closing stock of
Rs. 8,000 (Rs. 7,500¡¿Rs. 8,000/Rs. 33,000) 1,818
(iv) Total unrealised profit (Rs. 2,1000 + Rs. 1,818) 3,918
Q27. X Ltd., has a factory with two manufacturing Departments ¡®X¡¯ and ¡®Y¡¯. Part of the output of
Department X is transferred to Department Y for further processing and the balance is
directly transferred to selling Department. The entire production of Department Y is directly
transferred to the selling Department. Inter departmental stock transfers are made as follows :
X Department to Y Department at 33-1/3% over Departmental Cost.
X Department to selling department at 50% over Departmental Cost.
Y Department to selling department at 25% over Departmental Cost.
The following information is given for the year ended 31st March, 2008.
Particulars Dept. X Dept. Y Selling Dept.
Units Rs. Units Rs. Units Rs.
Opening Stock of Finished Goods 60 60,000 20 40,000 50 1,28,000
Opening Stock of Raw Materials . . . . .
Raw material Consumed . 1,82,000 . 20,000 . .
Labour Charges . 70,000 . 32,000 . .
Sales . . . . 120 4,80,000
Closing Stock of Finished Goods 40 . 50 . 60 .
52 Revisionary Test Paper (Revised Syllabus-2008)
Out of the total transfer by X Department, 30 units were transferred to selling department,
while the remaining to Department Y. The per unit material and labour consumption in X Department
on production to be transferred directly to selling department is 300 per cent of the labour and
material consumption on units transferred to Y Department. General Administration expenses Rs.
80,000.
Required : Prepare Departmental Profit and Loss Account and General Profit and Loss Account
for the year ended 31.3.2009.
Solution :
Departmental Profit and Loss Account
Dr. for the year ended 31st March, 2009 Cr.
X Dept. Y Dept. Selling Dept. X Dept. Y Dept. Sell Dept.
Particulars Qty. Rs. Qty. Rs. Qty. Rs. Particulars Qty. Rs. Qty. Rs. Qty. Rs.
To Opening stock 60 60,000 20 40,000 50 1,28,000 By Stock
To Raw Material 160 3,70,000 100 2,50,000 . .
consumed 1,82,000 . 20,000 . . By Sales . . . . 120 4,80,000
To Units By Closing
produced 140 . . . . . Stock 40 48,000 50 1,00,000 60 1,80,000
To Labour
Charges 70,000 . 32,000 . .
To Stock
Transferred
From X Dept. 130 2,08,000 30 1,62,00
To Stock
Transferred
From Y Dept. 100 2,50,000
To Departmental
Profit t/f to
General
P & L A/C 1,06,000 . 50,000 . 1,20,000
200 4,18,000 150 3,50,000 180 6,60,000 200 4,18,000 150 3,50,000 180 6,60,000
General Profit and Loss Account
Dr. for the year ended 31st March, 2008 Cr.
Particulars Rs. Particulars Rs.
To General Adm. Expenses 80,000 By Profit transferred from :
To Stock Reserve for Closing Stock X Dept. 1,06,000
on Dept. Y 12,000 Y Dept. 50,000
on Selling Dept. 18,175 Selling Dept. 1,20,000
To Net profit 1,65,825
2,76,000 2,76,000
Group-I : Paper-5 : Financial Accounting 53
Working Notes :
(a) Selling Department
Particulars Units Particulars Units
Opening Stock5 0 Sales 120
T/f from X Dept. 30 Closing Stock6 0
T/f from Y Dept. (Balancing figure) 100
180 180
(b) Y Department
Particulars Units Particulars Units
Opening Stock2 0 T/f to Selling Dept. 100
T/f from X Dept. 130 Closing Stock5 0
150 150
(c) X Department
Particulars Units Particulars Units
Opening Stock6 0 T/f to Selling Dept. 30
Production (Balancing figure) 140 T/f to Y Dept. 130
Closing Stock4 0
200 200
(d) Total Equivalent units produced in X Dept. in terms of those t/f to Y Dept.
= Equivalent units of those t/f to Sell Dept. + t/f to Y Dept. + Closing Stock.
= (30¡¿300/100) + 130 + 40 = 260
(e) Calculation of Transfer Prices and Closing Stock.
X Dept. Y Dept. Selling Dept.
Rs. Rs. Rs.
A Cost of Opening Stock 60,000 40,000 1,28,000
B Add : Cost of Raw Materials Consumed 1,82,000 20,000 .
C Add : Labour Charges 70,000 32,000 .
D Add : T/f from X Dept. . 2,08,000 1,62,000
E Add : T/f form Y Dept. . . 2,50,000
F Total Cost (A+B+C+D+E) 3,12,000 3,00,000 5,40,000
G Equivalant Units 260 150 180
H Average Cost per Equivalent Unit (F/G) 1,200 2,000 3,000
54 Revisionary Test Paper (Revised Syllabus-2008)
X Dept. Y Dept. Selling Dept.
Rs. Rs. Rs.
I Transfer Price of 130 Units t/f to Dept. Y
(a) Cost of 130 Units (130¡¿Rs. 1,200) 1,56,000
(b) Add : Profit element @ 33-1/3% 52,000
2,08,000
J Transfer Price of Units t/f to Selling Dept.
(a) Cost of Units t/f 1,08,000
(b) Add : Profit element 54,000 50,000
1,62,000 2,50,000
K Closing Stock48,000 1,00,000 1,80,000
(40¡¿Rs. 1,200) (50¡¿Rs. 2,000) (60¡¿Rs. 3,000)
(f) Unrealised Profit on Increase in Closing Stock of Y Dept. (Rs. 1,00,000 . Rs. 40,000)
A Current Cost incurred by Dept. Y
= Rs. 20,000 + Rs. 32,000 + Rs. 2,08,000 = Rs. 2,60,000
B Profit charged by Dept. X included in above (Rs. 2,08,000 ¡¿ 1/4) = Rs. 52,000
C Profit included in Increase in Closing Stock.
= (Rs. 52,000 ¡¿ Rs. 60,000/Rs. 2,60,000) = Rs. 12,000
(g) Profit Included in output transferred by Y Deptt. to Selling Dept.
A Transfer Price = Rs. 2,50,000
B Profit of Dept. Y included in Above (Rs. 2,50,000¡¿25/125)=Rs. 50,000
C Cost Element of Dept. X in Transfer Price (Rs. 2,50,000 . Rs. 50,000) =Rs. 2,00,000
D Profit of Dept. X included in above
(Rs. 2,00,000 ¡¿ Rs. 52,000/Rs. 2,60,000) = Rs. 40,000
E Total Profit Included in Transfer price (Rs. 50,000 + Rs. 40,000) = Rs. 90,000
(h) Profit Included in output transferred by X Dept. to Selling Dept.
= (Rs. 1,62,000 ¡¿ 50/150) = Rs. 54,000
(i) Total Profit included in output transferred to Selling Dept.
= Rs. 90,000 + Rs. 54,000 = Rs. 1,44,000
(j) Total Transfer Price for the Transfer made by X Dept. and Y Dept.
= Rs. 1,62,000 + Rs. 2,50,000 = Rs. 4,12,000
(k) Unrealised Profit included in increas in Closing Stock of Sell Dept.
= Rs. 1,44,000¡¿ Rs. 52,000/Rs. 4,12,000= Rs. 18,175
Group-I : Paper-5 : Financial Accounting 55
Q28. (a) From the following information find out the amount of provisions required to be made in
the Profit & Loss Account of a commercial bank for the year ended 31st March, 2009 :
(i) Packing credit outstanding from Food Processors Rs. 60 lakhs against which the bank
holds securities worth Rs. 15 lakhs. 40% of the above advance is covered by ECGC. The
above advance has remained doubtful for more than 3 years.
(ii) Other advances :
Assets classification Rs. in lakhs
Standard 3,000
Sub-standard 2,200
Doubtful :
For one year 900
For two years 600
For three years 400
For more than 3 years 300
Loss assets 600
Solution :
(i) (Rs. in lakhs)
Rs. Rs.
Amount outstanding (packing credit) 60
Less : Realisable value of securities 15
45
Less : ECGC cover (40%) 18
Balance 27
Required provision :
Provision for unsecured portion (100%) 27.0
Provision for secured portion (100%)* 15.0
42.0
(ii) Other advances :
(Rs. in lakhs)
Assets Amount % of Provision
Rs. provision Rs.
Standard 3,000 0.40* 12
Sub-standard 2,200 10 220
Doubtful :
For one year 900 20 180
For two years 600 30 180
For three years 400 30 120
For more than three years 300 100* 300
Loss 600 100 600
Required provision 1612
56 Revisionary Test Paper (Revised Syllabus-2008)
Note : Doubtful advances have been taken as fully secured. However, in case, the students assume
that no security cover is available for these advances, provision will be made for 100%.
* The above solution has been provided based on the latest NPA provisions (as per the Master
Circular issued by RBI ¡° DBOD No. BP. BC. 11/21.04.048/2005-06¡± dated November 4,
2005) though in the above question provisions for the year ended 31st march 2009 is
required.
Q29. (b) Bidisha Bank Ltd. had extended the following credit lines to a Small Scale Industry which
had not paid any interest since March, 2009
Term Loan Export Credit
Balance outstanding on 31.3.2001 Rs. 70 Lacs Rs. 60. Lacs
DICGC/ECGC Cover 50% 40%
Securities held Rs. 30 Lacs Rs. 25 Lacs
Realisable value of securities Rs. 20 Lacs Rs. 15 Lacs
Compute the necessary provisions to be made for the year ended 31st March, 2009.
Answer :
Term Loan Export Credit
Rs. in Lacs Rs. in Lacs
Balance outstanding 70.00 60.00
Less : Realisable value of securities 20.00 15.00
50.00 45.00
Less : DICGC/ECGC Cover 25.00 18.00
Net unsecured balance 25.00 27.00
Provision in respect of secured portion (100%)* 20.00
15.00
Provision for unsecured portion (100%) 25.00 27.00
Provision required 45.00 42.00
* The above solution has been provided based on the latest NPA provisions (as per the Master
Circular issued by RBI ¡° DBOD No. BP. BC. 11/21.04.048/2005-06¡± dated November 4, 2005)
though in the above question provisions for the year ended 31st March 2009 is required.
Q30. (a) Indian Insurance Co. Ltd. furnishes you with the following information :
(i) On 31.12.2008 it had reserve for unexpired risk to the tune of Rs. 40 crores. It comprised
of Rs. 15 crores in respect of marine insurance business : Rs. 20 crores in respect of fire
insurance business and Rs. 5 crores in respect of miscellaneous insurance business.
(ii) It is the practice of Indian Insurance Co. Ltd. to create reserves at 100% of net premium
income in respect of marine insurance policies and at 50% of net premium income in
respect of fire and miscellaneous income policies.
Group-I : Paper-5 : Financial Accounting 57
(iii) During 2008, the following business was conducted :
Marine Fire Miscellaneous
(Rs. in crores)
Rs. Rs. Rs.
Premia collected from :
(a) Insureds in respect of
policies issued 18 43 12
(b) Other insurance companies
in respect of risks undertaken 7 5 4
Premia paid/payable to other insurance
companies on business ceded 6.7 4.3 7
Indian Insurance Co. Ltd. asks you to :
(a) Pass journal entries relating to ¡°Unexpired risks reserve¡±.
(b) Show in columnar form ¡°Unexpired risks reserve¡± a/c for 2008
Answer : (a) Journal of Indian Insurance Co. Ltd.
(Rupees in crores)
Dr. Cr.
Rs Rs.
2008
Dec. 31 Marine Revenue A/c Dr. 3.30
To Unexpired Risks Reserve A/c 3.30
(Being the difference between closing provision of
Rs. 18.30 crores (18 + 7 . 6.7) and opening provision
of Rs. 15 crores charged to marine revenue account)
Fire Revenue A/c Dr. 1.85
To Unexpired Risks Reserve A/c 1.85
(Being the difference between closing provision of
Rs. 21.85 crores [(43 + 5 . 4.3)/2] and opening
provision of Rs. 20 crores charged to fire revenue
account)
Unexpired Risks Reserve A/c Dr. 0.50
To Miscellaneous Revenue A/c 0.50
(Being the excess of opening balance of Rs. 5 crores
over the required closing balance of Rs. 4.5 crores
[(12 + 4 . 7)/2] credited to miscellaneous revenue account).
58 Revisionary Test Paper (Revised Syllabus-2008)
(b) Unexpired Risks Reserve A/c
(Rs in crores)
Marine Fire Miscel- Marine Fire Miscellaneous
laneous
1997 Rs. Rs. Rs. 1997 Rs. Rs. Rs.
Dec. 31 To Revenue A/c . . 0.50 Jan 1 By Balance b/d 15.00 20.00 5.00
To Balance c/d 18.30 21.85 4.50 Dec. 31 By Revenue A/c 3.30 1.85 .
18.30 21.85 5.00 18.30 21.85 5.00
Note : Alternatively, the opening balances of unexpired risk reserves may be reversed in the
beginning of year by transfer to Revenue account and fresh reserve of full required amount
may be created at the end of the year which will be carried forward as closing balances.
Q30. (b) From the following figures appearing in the books of Fire Insurance division of a General
Insurance Company, show the amount of claim as it would appear in the Revenue Account
for the year ended 31st March, 2009 :
Direct Business Re-Insurance
Rs. Rs.
Claim paid during the year 46,70,000 7,00,000
Claim Payable. 1st April, 2008 7,63,000 87,000
31st March, 2009 8,12,000 53,000
Claims received . 2,30,000
Claims Receivable. 1st April, 2008 . 65,000
31st March, 2009 . 1,13,000
Expenses of Management 2,30,000 .
(includes Rs. 35,000 Surveyor¡¯s fee and Rs. 45,000
Legal expenses for settlement of claims)
Solution :
General Insurance Company
(Abstract showing the amount of claims)
Rs. ¡¯000 Rs. ¡¯000
Claims less Re-insurance :
Paid during the year 52,20
Add : Outstanding claims at the end of the year 7,52
59,72
Less : Outstanding claims at the beginning of the year 7,85 51,87
Group-I : Paper-5 : Financial Accounting 59
Working Notes :
Rs. ¡¯000 Rs. ¡¯000
1. Claims paid during the year
Direct business 46,70
Reinsurance 7,00 53,70
Add : Surveyor¡¯s fee 35
Legal expenses 45 80
54,50
Less : Claims received from re-insurers 2,30
52,20
2. Claims outstanding on 31st March, 2009
Direct business 8,12
Reinsurance 53 8,65
Less : Claims receivable from re-insurers 1,13
7,52
3. Claims outstanding on 1st April, 2008
Direct business 763
Reinsurance 87 8,50
Less : Claims receivable from re-insurers 65
7,85
Q30. (c) Heaven Life Insurance Co. furnishes you the following information:
Rs.
Life Insurance fund on 31.3.2009 52,00,000
Net liability on 31.3.2009 as per actuarial valuation 40,00,000
Interim bonus paid to policyholders during intervaluation period 3,00,000
You are required to prepare:
(i) Valuation Balance Sheet;
(ii) Statement of Net Profit for the valuation period; and
(iii) Amount due to the policyholders.
Solution :
(i) Heaven Life Insurance Co.
Valuation Balance Sheet as at 31st March, 2009
Rs. Rs.
To Net Liability as per actuarial valuation 40,00,000 By Life Assurance Fund 52,00,000
To Surplus 12,00,000
52,00,000 52,00,000
(ii) Statement showing Net Profit for the valuation period
Rs.
Surplus as per Balance Sheet (i.e., Valuation Balance Sheet) 12,00,000
Add:Interim bonus paid 3,00,000
15,00,000
60 Revisionary Test Paper (Revised Syllabus-2008)
(iii) Amount due to policyholders
Rs.
95% of net profit due to policyholders (95% of Rs. 15,00,000) 14,25,000
Less : Interim bonus already paid 3,00,000
Amount due to policyholders 11,25,000
Q31. State the main features of ¡°Double Accounts¡± system of presentation of financial information
in the case of public utility concern.
Answer :
Double accounts system is the name given to the system of preparing the final accounts of certain
statutory companies formed by special Acts of parliament, usually public utility undertakings (for
example Electricity Companies). The double accounts system is not a special method of keeping
accounts, rather a special method of presenting accounts which are kept under the normal double
entry system. Under this system, separate accounts in respect of capital and revenue are prepared
in order to show clearly the capital receipts and the manner in which the amounts thereof have
been invested. The final accounts prepared under the double accounts system normally consist
of :
(i) Revenue Account
(ii) Net Revenue Account
(iii) Capital Account (Receipts and Expenditure on capital account)
(iv) General Balance Sheet.
The Revenue account is analogous to the Profit & Loss Account of a company with some exceptions.
The Net Revenue Account resembles with appropriation portion of the Profit & Loss Account of a
company. The Balance Sheet is presented in two parts namely Capital Account and General
Balance Sheet. The Capital Account shows the total amount of capital raised and its sources and
also the manner and extent to which this capital has been applied in the acquisition of fixed assets
for the purpose of carrying on the business. The General balance sheet includes the other items.
The Double accounts system in its pure form does no longer exist but the statements submitted to
State Governments by electricity companies generally follow the principle of double accounts
system. It may be noted that for presenting accounts to the shareholders, electricity companies
normally follow Schedule VI of the Companies Act, 1956.
Q32. Electric Supply Ltd. rebuilt and re-equipped one of their Mains at a Cash Cost of Rs. 40,00,000.
The old Mains thus superseded cost Rs. 15,00,000. The capacity of the new Main is double
that of the old Main.
Rs. 70,000 was realised from sale of old materials. Four old motors valued at Rs. 2,00,000
salvaged from the old Main were used in the reconstruction. The cost of Labour and Materials
is respectively 30% and 25% higher now than when the old Main was built. The proportion
of Labour to Materials in the Main then and now is 2 : 3.
Show the Journal entries for recording the above transactions, if accounts are maintained
under Double Account System.
Group-I : Paper-5 : Financial Accounting 61
Solution:
Electric Supply Ltd.
Journal Entries
Dr. Cr.
Rs. Rs.
New Main Account Dr. 20,95,000
Replacement Account Dr. 19,05,000
To Bank Account 40,00,000
(Being current cost of replacement charged to replacement
account and the balance amount capitalised)
New Main Account Dr. 2,00,000
To Replacement Account 2,00,000
(Being the value of motors salvaged from old main
used in the reconstruction of main)
Bank Account Dr. 70,000
To Replacement Account 70,000
(Being the amount realised from sale of old materials
credited to replacement account)
Revenue Account Dr. 16,35,000
To Replacement Account 16,35,000
(Being the net current cost of replacement
transferred to revenue account)
Working Notes :
1. Current cost of replacement:
Cost of Increase in cost Current
existing main Rate Amount cost
Rs. Rs. Rs.
Materials (3/5 ¡¿ Rs. 15 lacs) 9,00,000 25% 2,25,000 11,25,000
Labour (2/5 ¡¿ Rs. 15 lacs) 6,00,000 30% 1,80,000 7,80,000
Estimated current cost for replacement
of present main (amount to be charged
to replacement account) 19,05,000
2. Additional cost of reconstruction of main (to be capitalised)
Cash cost of re-building new main 40,00,000
Less: Estimated current cost for replacement of existing old main 19,05,000
Additional cost in new main to be capitalised (excluding old motors used 20,95,000
62 Revisionary Test Paper (Revised Syllabus-2008)
3. Replacement Account
Dr. Cr.
To Bank A/c 19,05,000 By New Main A/c 2,00,000
By Bank A/c 70,000
By Replacement A/c
(Balancing figure) 16,35,000
19,05,000 19,05,000
Reasonable returns in electricity supply companies.
Answer :
The law seeks to prevent an electricity undertaking from earning too high a profit. For this purpose,
¡°reasonable return¡± has been defined as consisting of :
(a) An yield at the standard rate which is Reserve Bank rate plus two percent on the capital
base as defined below;
(b) Income derived from investment except investment made against Contingencies Reserve;
(c) An amount equal to 1/2% on loans advanced by the Electricity Board;
(d) An amount equal to 1/2% on the amounts borrowed from organisations or institutions
approved by the State Government;
(e) An amount equal to 1/2% on the amount raised by the issue of debentures;
(f) An amount equal to 1/2% on balance of Development Reserve; and
(g) Such other amounts as may be allowed by the Central Government having regard to the
prevailing tax structure in the country.
The term ¡°Capital Base¡± used above, can be defined as:
(a) the original cost of fixed assets available for use and necessary for the purpose of the
undertaking less contributions, if any made by the consumers for construction of service
lines and also amounts written off;
(b) the cost of intangible assets;
(c) the original cost of work in progress;
(d) the amount of investments compulsorily made agaisnt contingencies reserve; and
(e) the monthly average of the stores, materials, supplies and cash and bank balances held at
the end of each month of the year of account not exceeding in the aggregate an amount
equal to one quarter of the expenditure.
Less:
(i) the amount written off or set aside on account of depreciation of fixed assets and amounts
written off in respect of intangible assets in the books of the undertaking;
(ii) the amount of any loans advanced by the Board;
(iii) the amount of any loans borrowed from organisations or institutions approved by the State
Government;
(iv) the amount of any debentures issued by the licensee;
Group-I : Paper-5 : Financial Accounting 63
(v) the amount of security deposits held in cash;
(vi) the amount standing to the credit of the Tariffs and Dividends Control Reserve;
(vii) the amount set apart for the development reserve; and
(viii) the amount carried forward in the accounts of the licensee for distribution to the consumers.
Q33. From the following information, prepare a summarised Balance Sheet as at 31st March,
2009 :
Working Capital Rs. 2,40,000
Bank overdraft Rs. 40,000
Fixed Assets to Proprietary ratio 0.75
Reserves and Surplus Rs. 1,60,000
Current ratio 2.5
Liquid ratio 1.5
Working notes :
1. Current assets and Current liabilities computation :









=









= or






= = k (say)
Or Current assets = 2.5 k and Current liabilities = k
Or Working capital = ( Current assets . Current liabilities)
Or Rs. 2,40,000 = k (2.5 . 1) = 1.5 k
Or Rs. 1,60 ,000 k=
¡Å Current liabilities = Rs. 1,60,000
Current assets = Rs. 1,60,000 ¡¿ 2.5 = Rs. 4,00,000
2. Computation of stock
Liquid ratio=





Or 1.5 =


.

Or 1.5 ¡¿ Rs.1,60,000=Rs.4,00,000 . Stock
Or Stock= Rs.1,60,000
64 Revisionary Test Paper (Revised Syllabus-2008)
3. Computation of Proprietary fund; Fixed assets; Capital and Sundry creditors
Proprietary ratio =





=
Fixed assets = 0.75 Proprietary fund
and Net working capital = 0.25 Proprietary fund
Or Rs. 2,40,000/0.25 = Proprietary fund
Or Proprietary fund = Rs. 9,60,000
and Fixed assets = 0.75 proprietary fund
= 0.75 ¡¿ Rs. 9,60,000
= Rs. 7,20,000
Capital = Proprietary fund - Reserves & Surplus
= Rs. 9,60,000 . Rs. 1,60,000
= Rs. 8,00,000
Sundry creditors = (Current liabilities . Bank overdraft)
= (Rs.1,60,000 . Rs.40,000)
= Rs.1,20,000
Balance Sheet
Rs. Rs.
Capital 8,00,000 Fixed assets 7,20,000
Reserves & Surplus 1,60,000 Stock1 ,60,000
Bank overdraft 40,000 Current assets 2,40,000
Sundry creditors 1,20,000
11,20,000 11,20,000
Q33. (b) Discuss any three ratios computed for investment analysis.
Answer:
Three ratios computed for investment analysis are as follows;
(i) Earning per share =




Group-I : Paper-5 : Financial Accounting 65
(ii) Dividend yield ratio =


¡¿


(iii) Return on capital employed =














Q33. (c) Discuss the financial ratios for evaluating company performance on operating efficiency
and liquidity position aspects.
Answer :
Financial ratios for evaluating performance on operational efficiency and liquidity position aspects
are discussed as:
Operating Efficiency: Ratio analysis throws light on the degree of efficiency in the management
and utilization of its assets. The various activity ratios (such as turnover ratios) measure this kind
of operational efficiency. These ratios are employed to evaluate the efficiency with which the
firm manages and utilises its assets. These ratios usually indicate the frequency of sales with
respect to its assets. These assets may be capital assets or working capital or average inventory.
In fact, the solvency of a firm is, in the ultimate analysis, dependent upon the sales revenues
generated by use of its assets . total as well as its components.
Liquidity Position : With the help of ratio analysis, one can draw conclusions regarding liquidity
position of a firm. The liquidity position of a firm would be satisfactory, if it is able to meet its
current obligations when they become due. Inability to pay-off short-term liabilities affects its
credibility as well as its credit rating. Continuous default on the part of the business leads to
commercial bankruptcy. Eventually such commercial bankruptcy may lead to its sickness and
dissolution. Liquidity ratios are current ratio, liquid ratio and cash to current liability ratio. These
ratios are particularly useful in credit analysis by banks and other suppliers of short-term loans.
Q34. (a) Completed Contract Method.
Answer :
It is a revenue recognition method for long term construction contracts under which revenue is
recognized only when the contract is completed or substantially completed; i.e., when only minor
work is expected to complete the contract other than warranty obligations. Costs and progress
payments received are accumulated during the course of contract but revenue is not recognized
until the contract activity is substantially completed.
However, it is necessary to create provision for anticipated losses for the unfinished contract
work although revenue is recognized only when the contract is substantially completed.
The principal advantage of this method is that it is based on results as determined when the
contract is completed or substantially completed rather than on estimates which may require
subsequent adjustments as a result of unforeseen cost and possible losses. If this method is followed,
the risk of recognizing profit which is not really earned is greatly minimised.
The disadvantage is that periodic reported income does not reflect the level of activity on contracts
during the period.
66 Revisionary Test Paper (Revised Syllabus-2008)
Q34. (b) Discuss the Computer Software Accounting.
Answer:
In case computer software is acquired for sale in the ordinary course of business, it is treated as
inventory and AS-2, Valuation of Inventories, is applicable which has been discussed in Section I
of the book. If it is meant for internal use, AS-26 is applicable.
Computer software for internal use can be either:
(i) acquired, or
(ii) internally generated.
SOFTWARE ACQUIRED FOR INTERNAL USE
The cost of a software acquired for internal use should be recognized as an asset if it meets the
recognition criteria prescribed in paragraphs 20 and 21 of the Accounting Standard-26* of the
Institute of Chartered Accountants of India which are given below:
¡° The recognition of an item as an intangible asset requires an enterprise to demonstrate that the
item meets the following definition and criteria¡±

Comments (0)

Post a Comment